Maternity Test Questions

Pataasin ang iyong marka sa homework at exams ngayon gamit ang Quizwiz!

2. A pregnant woman with diabetes at 10 weeks' gestation has a glycosylated hemoglobin (HbA1c) level of 13%. At this time the nurse should be most concerned about which possible fetal outcome? A. congenital anomalies B. incompetent cervix C. placenta previa D. placental abruption (abruptio placentae)

Answer: A Rationale: A HbA1c level of 13% indicates poor glucose control. This, in conjunction with the woman being in the first trimester, increases the risk for congenital anomalies in the fetus. Elevated glucose levels are not associated with incompetent cervix, placenta previa, or placental abruption (abruptio placentae).

28. A pregnant client at 24 weeks' gestation comes to the clinic for an evaluation. The client called the clinic earlier in the day stating that she had not felt the fetus moving since yesterday evening. Further assessment reveals absent fetal heart tones. Intrauterine fetal demise is suspected. The nurse would expect to prepare the client for which testing to confirm the suspicion? A. Ultrasound B. Amniocentesis C. Human chorionic gonadotropin (hCG) level D. Triple marker screening

Answer: A Rationale: A client experiencing an intrauterine fetal demise (IUFD) is likely to seek care when she notices that the fetus is not moving or when she experiences contractions, loss of fluid, or vaginal bleeding. History and physical examination frequently are of limited value in the diagnosis of fetal death, since many times the only history tends to be recent absence of fetal movement and no fetal heart beat heard. An inability to obtain fetal heart sounds on examination suggests fetal demise, but an ultrasound is necessary to confirm the absence of fetal cardiac activity. Once fetal demise is confirmed, induction of labor or expectant management is offered to the client. An amniocentesis, hCG level, or triple marker screening would not be used to confirm IUFD.

8. A client comes to the clinic for an evaluation. The client is at 22 weeks' gestation. After reviewing a client's history, which factor would the nurse identify as placing her at risk for preeclampsia? A. Her mother had preeclampsia during pregnancy. B. Client has a twin sister. C. Her sister-in-law had gestational hypertension. D. This is the client's second pregnancy.

Answer: A Rationale: A family history of preeclampsia, such as a mother or sister, is considered a risk factor for the client. Having a twin sister or having a sister-in-law with gestational hypertension would not increase the client's risk. If the client had a history of preeclampsia in her first pregnancy, then she would be at risk in her second pregnancy.

1. After teaching a woman who has had an evacuation for gestational trophoblastic disease (hydatidiform mole or molar pregnancy) about her condition, which statement indicates that the nurse's teaching was successful? A. "I will be sure to avoid getting pregnant for at least 1 year." B. "My intake of iron will have to be closely monitored for 6 months." C. "My blood pressure will continue to be increased for about 6 more months." D. "I won't use my birth control pills for at least a year or two."

Answer: A Rationale: After evacuation of trophoblastic tissue (hydatiform mole), long-term follow-up is necessary to make sure any remaining trophoblastic tissue does not become malignant. Serial hCG levels are monitored closely for 1 year, and the client is urged to avoid pregnancy for 1 year because it can interfere with the monitoring of hCG levels. Iron intake and blood pressure are not important aspects of follow up after evacuation of a hydatiform mole. Use of a reliable contraceptive is strongly recommended so that pregnancy is avoided.

6. A client experienced prolonged labor with prolonged premature rupture of membranes. The nurse would be alert for which condition in the mother and the newborn? A. infection B. hemorrhage C. trauma D. hypovolemia

Answer: A Rationale: Although hemorrhage, trauma, and hypovolemia may be problems, the prolonged labor with the prolonged premature rupture of membranes places the client at high risk for a postpartum infection. The rupture of membranes removes the barrier of amniotic fluid, so bacteria can ascend.

23. A nurse is teaching a woman about measures to prevent preterm labor in future pregnancies because the woman just experienced preterm labor with her most recent pregnancy. The nurse determines that the teaching was successful based on which statement by the woman? A. "I'll make sure to limit the amount of long distance traveling I do." B. "Stress isn't a problem that is related to preterm labor." C. "Separating pregnancies by about a year should be helpful." D. "I'll need extra iron in my diet so I have extra for the baby."

Answer: A Rationale: Appropriate measures to reduce the risk for preterm labor include: avoiding travel for long distances in cars, trains, planes or buses; achieving adequate iron store through balanced nutrition (excess iron is not necessary); waiting for at least 18 months between pregnancies, and using stress management techniques for stress.

7. After the birth of a newborn, which action would the nurse do first to assist in thermoregulation? A. Dry the newborn thoroughly. B. Put a hat on the newborn's head. C. Check the newborn's temperature. D. Wrap the newborn in a blanket.

Answer: A Rationale: Drying the newborn immediately after birth using warmed blankets is essential to prevent heat loss through evaporation. Then the nurse would place a cap on the baby's head and wrap the newborn. Assessing the newborn's temperature would occur once these measures were initiated to prevent heat loss.

21. A pregnant woman with gestational diabetes comes to the clinic for a fasting blood glucose level. When reviewing the results, the nurse determines that the woman is achieving good glucose control based on which result? A. 88 mg/dL B. 100 mg/dL C. 110 mg/dL D. 120 mg/dL

Answer: A Rationale: For a pregnant woman with diabetes, the ADA and ACOG recommend maintaining a fasting blood glucose level below 95 mg/dL, with postprandial levels below 140 mg/dL at 1 hour, below 120 mg/dL at 2 hours.

6. A client expresses concern that her 2-hour-old newborn is sleepy and difficult to awaken. The nurse explains that this behavior indicates: A. normal progression of behavior. B. probable hypoglycemia. C. physiological abnormality. D. inadequate oxygenation.

Answer: A Rationale: From 30 to 120 minutes of age, the newborn enters the second stage of transition, that of sleep or a decrease in activity. More information would be needed to determine if hypoglycemia, a physiologic abnormality, or inadequate oxygenation was present.

28. A nurse is teaching a group of new parents about their newborns' sensory capabilities. The nurse would identify which sense as being well-developed at birth? A. hearing B. touch C. taste D. vision

Answer: A Rationale: Hearing is well developed at birth, evidenced by the newborn's response to noise by turning. Vision is the least mature sense at birth. Touch is evidenced by the newborn's ability to respond to tactile stimuli and pain. A newborn can distinguish between sweet and sour by 72 hours of age.

19. A pregnant woman comes to the clinic for her first evaluation. The woman is screened for hepatitis B (HBV) and tests positive. The nurse would anticipate administering which agent? A. HBV immune globulin B. HBV vaccine C. acylcovir D. valacyclovir

Answer: A Rationale: If a woman tests positive for HBV, expect to administer HBV immune globulin. The newborn will also receive HBV vaccine within 12 hours of birth. Acyclovir or valacyclovir would be used to treat herpes simplex virus infection.

1. A newborn with severe meconium aspiration syndrome (MAS) is not responding to conventional treatment. Which measure would the nurse anticipate as possibly necessary for this newborn? A. extracorporeal membrane oxygenation (ECMO) B. respiratory support with a ventilator C. insertion of a laryngoscope for deep suctioning D. replacement of an endotracheal tube via X-ray

Answer: A Rationale: If conventional measures are ineffective, then the nurse would need to prepare the newborn for ECMO. Hyperoxygenation, ventilatory support, and direct tracheal suctioning are typically used initially to promote tissue perfusion. However, if these are ineffective, ECMO would be the next step.

18. A nurse is preparing a teaching program for a group of pregnant women about preventing infections during pregnancy. When describing measures for preventing cytomegalovirus infection, which measure would the nurse include as a priority? A. frequent handwashing B. immunization C. prenatal screening D. antibody titer screening

Answer: A Rationale: Most women are asymptomatic and do not know they have been exposed to CMV. Prenatal screening for CMV infection is not routinely performed. Since there is no therapy that prevents or treats CMV infections, nurses are responsible for educating and supporting childbearing-age women at risk for CMV infection. Stressing the importance of good handwashing and use of sound hygiene practices can help to reduce transmission of the virus. There is no immunization for CMV. Antibody titer levels would be useful for identifying women at risk for rubella.

15. A new mother asks the nurse, "Why has my baby lost weight since he was born?" The nurse integrates knowledge of which cause when responding to the new mother? A. insufficient calorie intake B. shift of water from extracellular space to intracellular space C. increase in stool passage D. overproduction of bilirubin

Answer: A Rationale: Normally, term newborns lose 5% to 10% of their birth weight as a result of insufficient caloric intake within the first week after birth, shifting of intracellular water to extracellular space, and insensible water loss. Stool passage and bilirubin have no effect on weight loss.

5. When teaching a class of pregnant women about the effects of substance use during pregnancy, the nurse would include which effect? A. low-birthweight infants B. excessive weight gain C. higher pain tolerance D. longer gestational periods

Answer: A Rationale: Substance use during pregnancy is associated with low birth weight infants, preterm labor, abortion, intrauterine growth restriction, abruptio placentae, neurobehavioral abnormalities, and long-term childhood developmental consequences. Excessive weight gain, higher pain tolerance, and longer gestational periods are not associated with substance use.

5. A multipara client develops thrombophlebitis after birth. Which assessment findings would lead the nurse to intervene immediately? A. dyspnea, diaphoresis, hypotension, and chest pain B. dyspnea, bradycardia, hypertension, and confusion C. weakness, anorexia, change in level of consciousness, and coma D. pallor, tachycardia, seizures, and jaundice

Answer: A Rationale: Sudden unexplained shortness of breath and reports of chest pain along with diaphoresis and hypotension suggest pulmonary embolism, which requires immediate action. Other signs and symptoms include tachycardia, apprehension, hemoptysis, syncope, and sudden change in the woman's mental status secondary to hypoxemia. Anorexia, seizures, and jaundice are unrelated to a pulmonary embolism.

13. The nurse is providing care to several pregnant women who may be scheduled for labor induction. The nurse identifies the woman with which Bishop score as having the best chance for a successful induction and vaginal birth? A. 11 B. 7 C. 5 D. 13

Answer: A Rationale: The Bishop score helps identify women who would be most likely to achieve a successful induction. The duration of labor is inversely correlated with the Bishop score: a score over 8 indicates a successful vaginal birth. Therefore the woman with a Bishop score of 11 would have the greatest chance for success. Bishop scores of less than 6 usually indicate that a cervical ripening method should be used prior to induction.

7. A nurse is developing the plan of care for a small-for-gestational-age newborn. Which action would the nurse determine as a priority?A. Preventing hypoglycemia with early feedings B. Observing for newborn reflexes C. Promoting bonding between the parents and the newborn D. Monitoring vital signs every 2 hours

Answer: A Rationale: The nurse must consider the implications of a small-for-gestational-age newborn. With the loss of the placenta at birth, the newborn must now assume control of glucose homeostasis. This is achieved by early oral intermittent feedings. Observing for newborn reflexes, promoting bonding, and monitoring vital signs, although important, are not the priority for this newborn.

23. While changing a female newborn's diaper, the nurse observes a mucus-like, slightly bloody vaginal discharge. Which action would the nurse do next? A. Document this as pseudo menstruation B. Notify the primary care provider immediately. C. Obtain a culture of the discharge. D. Inspect for engorgement.

Answer: A Rationale: The nurse should assess pseudomenstruation, a vaginal discharge composed of mucus mixed with blood, which may be present during the first few weeks of life. This discharge requires no treatment. The discharge is a normal finding and thus does not need to be reported immediately. It is not an indication of infection. The female genitalia normally will be engorged, so assessing for engorgement is not indicated.

18. The nurse is inspecting the external genitalia of a male newborn. Which finding would alert the nurse to a possible problem? A. limited rugae B. large scrotum C. palpable testes in scrotal sac D. negative engorgement

Answer: A Rationale: The scrotum usually appears relatively large and should be pink in white neonates and dark brown in neonates of color. Rugae should be well formed and should cover the scrotal sac. There should not be bulging, edema(engorgement), or discoloration. Testes should be palpable in the scrotal sac and feel firm and smooth and be of equal size on both sides of the scrotal sac.

4. Assessment of a newborn reveals a heart rate of 180 beats per minute. To determine whether this finding is a common variation rather than a sign of distress, what else does the nurse need to know? A. How many hours old is this newborn? B. How long ago did this newborn eat? C. What was the newborn's birthweight? D. Is acrocyanosis present?

Answer: A Rationale: The typical heart rate of a newborn ranges from 110 to 160 beats per minute with wide fluctuation during activity and sleep. Typically heart rate is assessed every 30 minutes until stable for 2 hours after birth. The time of the newborn's last feeding and his birthweight would have no effect on his heart rate. Acrocyanosis is a common normal finding in newborns.

7. When assessing the postpartum woman, the nurse uses indicators other than pulse rate and blood pressure for postpartum hemorrhage because: A. these measurements may not change until after the blood loss is large. B. the body's compensatory mechanisms activate and prevent any changes. C. they relate more to change in condition than to the amount of blood lost. D. maternal anxiety adversely affects these vital signs.

Answer: A Rationale: The typical signs of hemorrhage do not appear in the postpartum woman until as much as 1,800 to 2,100 ml of blood has been lost. In addition, accurate determination of actual blood loss is difficult because of blood pooling inside the uterus and on perineal pads, mattresses, and the floor.

18. A pregnant woman gives birth to a small for gestational age neonate who is admitted to the neonatal intensive care unit with seizure activity. The neonate appears to have abnormally small eyes and a thin upper lip. The infant is noted to be microcephalic. Based on these findings, which substance would the nurse suspect the women of using during pregnancy? A. alcohol B. cocaine C. heroin D. methamphetamine

Answer: A Rationale: This child's features match those of fetal alcohol syndrome, including microcephaly, small palpebral (eyelid) fissures, abnormally small eyes, and fetal growth restriction.

30. A newborn is exhibiting symptoms of withdrawal and toxicology test have been prescribed. Which type of specimen should the nurse collect to obtain the most accurate results? A. Meconium B. Blood C. Urine D. Sputum

Answer: A Rationale: Toxicology screening of a newborn can include testing from blood, urine and meconium. These tests identify which drugs the newborn has been exposed to in utero. A meconiumsamplecandetectwhichdrugsthemotherhasbeenusing fromthesecondtrimester of pregnancy until birth. It is the preferred method of testing. A urine screen identifies only the drugs the mother has used recently. The nurse should be careful not to mix the meconium sample with urine as it alters the results of the test. Blood samples can be taken and they will yield results of current drugs in the newborn's system, but they are invasive and noninvasive testing will provide the same results. If possible, testing on the mother is preferred. This testing provides quick results of what drugs the mother has been exposing the fetus to in utero. This will help in planning and providing care for the drug-exposed newborn. Sputum is not used for toxicology studies.

6. A preterm newborn has received large concentrations of oxygen therapy during a 3-month stay in the NICU. As the newborn is prepared to be discharged home, the nurse anticipates a referral for which specialist? A. ophthalmologist B. nephrologist C. cardiologist D. neurologist

Answer: A Rationale: Use of large concentrations of oxygen and sustained oxygen saturations higher than 95% while on supplemental oxygen have been associated with the development of retinopathy of prematurity (ROP) and further respiratory complications in the preterm newborn (Martin & Deakins, 2020). For these reasons, oxygen should be used judiciously to prevent the development of further complications. A guiding principle for oxygen therapy is it should be targeted to levels appropriate to the condition, gestational age, and postnatal age of the newborn. As a result, an ophthalmology consult for follow-up after discharge is essential for preterm infants who have received extensive oxygen. Although referrals to other specialists may be warranted depending on the newborn's status, there is no information to suggest that any would be needed.

5. Which action would be most appropriate for the nurse to take when a newborn has an unexpected anomaly at birth? A. Show the newborn to the parents as soon as possible while explaining the defect. B. Remove the newborn from the birthing area immediately. C. Inform the parents that there is nothing wrong at the moment. D. Tell the parents that the newborn must go to the nursery immediately.

Answer: A Rationale: When an anomaly is identified at or after birth, parents need to be informed promptly and given a realistic appraisal of the severity of the condition, the prognosis, and treatment options so that they can participate in all decisions concerning their child. Removing the newborn from the area or telling them that the newborn needs to go to the nursery immediately is inappropriate and would only add to the parents' anxieties and fears. Telling them that nothing is wrong is inappropriate because it violates their right to know.

25. A pregnant client has received dinoprostone. Following administration of this medication, the nurse assesses the client and determines that the client is experiencing an adverse effect of the medication based on which client report? Select all that apply. A. headache B. nausea C. diarrhea D. tachycardia E. hypotension

Answer: A, B, C Rationale: Adverse effects associated with dinoprostone include headache, nauseas and vomiting, and diarrhea. Tachycardia and hypotension are not associated with this drug.

15. A postpartum woman is diagnosed with endometritis. The nurse interprets this as an infection involving which area? Select all that apply. A. endometrium B. decidua C. myometrium D. broad ligament E. ovaries F. fallopian tubes

Answer: A, B, C Rationale: Endometritis is an infectious condition that involves the endometrium, decidua, and adjacent myometrium of the uterus. Extension of endometritis can result in parametritis, which involves the broad ligament and possibly the ovaries and fallopian tubes, or septic pelvic thrombophlebitis.

20. After teaching a pregnant woman with iron deficiency anemia about nutrition, the nurse determines that the teaching was successful when the woman identifies which foods as being good sources of iron in her diet? Select all that apply. A. dried fruits B. peanut butter C. meats D. milk E. white bread

Answer: A, B, C Rationale: Foods high in iron include meats, green leafy vegetables, legumes, dried fruits, whole grains, peanut butter, bean dip, whole-wheat fortified breads, and cereals.

21. A premature, 36-week-gestation neonate is admitted to the observational nursery and placed under bili-lights with evidence of hyperbilirubinemia. Which assessment findings would the neonate demonstrate? Select all that apply. A. increased serum bilirubin levels B. clay-colored stools C. tea-colored urine D. cyanosis E. Mongolian spots

Answer: A, B, C Rationale: Hyperbilirubinemia is indicated when the newborn presents with elevated serum bilirubin levels, tea-colored urine, and clay-colored stools. Cyanosis would not be seen in infants in this scenario. Mongolian spots are not associated with newborn jaundice.

25. A preterm newborn is admitted to the neonatal intensive care with the diagnosis of an omphalocele. What nursing actions would the nurse perform? Select all that apply. A. The abdominal contents are protected. B. Fluid loss of the neonate will be minimized. C. Perfusion to the exposed abdominal contents will be maintained. D. Neonatal weight loss will be prevented. E. Assessment of hyperbilirubinemia will be monitored.

Answer: A, B, C Rationale: Nursing management of newborns with omphalocele or gastroschisis focuses on preventing hypothermia, maintaining perfusion to the eviscerated abdominal contents by minimizing fluid loss, and protecting the exposed abdominal contents from trauma and infection. Weight loss at this point is not a priority, and neither is assessing bilirubin.

27. A nurse is taking a history on a woman who is at 20 weeks' gestation. The woman reports that she feels some heaviness in her thighs since yesterday. The nurse suspects that the woman may be experiencing preterm labor based on which additional assessment findings? Select all that apply. A. dull low backache B. viscous vaginal discharge C. dysuria D. constipation E. occasional cramping

Answer: A, B, C Rationale: Symptoms of preterm labor are often subtle and may include change or increase in vaginal discharge with mucus, water, or blood in it; pelvic pressure; low, dull backache; nausea, vomiting or diarrhea, and heaviness or aching in the thighs. Constipation is not known to be a sign of preterm labor. Preterm labor is assessed when there are more than six contractions per hour. Occasional asymptomatic cramping can be normal.

23. A 33 weeks' gestation neonate is being assessed for necrotizing enterocolitis (NEC). Which nursing actions would the nurse implement? Select all that apply. A. Perform hemoccult tests on stools. B. Monitor abdominal girth. C. Measure gastric residual before feeds. D. Assess bowel sounds before each feed. E. Assess urine output.

Answer: A, B, C, D Rationale: Always keep the possibility of NEC in mind when dealing with preterm newborns, especially when enteral feedings are being administered. Note feeding intolerance, diarrhea, bile- stained emesis, or grossly bloody stools. Perform hemoccult tests on the bloody stool. Assess the neonate's abdomen for distention, tenderness, and visible loops of bowel. Measure the abdominal circumference, noting an increase. Listen to bowel sounds before each feeding. Determine residual gastric volume prior to feeding; when it is elevated, be suspicious for NEC. Assessing urine output is not essential.

26. The nurse in the neonatal intensive care unit is caring for a neonate she suspects is at risk for an intraventricular hemorrhage (IVH). Which nursing actions would be priorities? Select all that apply. A. Maintain fetal flexed position. B. Administer fluids slowly. C. Assess for bulging fontanel .D. Measure head circumference daily. E. Assess Moro reflex. F. Measure intake and output.

Answer: A, B, C, D Rationale: Care of the newborn with IVH is primarily supportive. Correct anemia, acidosis, and hypotension with fluids and medications. Administer fluids slowly to prevent fluctuations in blood pressure. Avoid rapid volume expansion to minimize changes in cerebral blood flow. Keep the newborn in a flexed, contained position with the head elevated to prevent or minimize fluctuations in intracranial pressure. Continuously monitor the newborn for signs of hemorrhage, such as changes in the level of consciousness, bulging fontanel, seizures, apnea, and reduced activity level. Also, measuring head circumference daily to assess for expansion in size is essential in identifying complications early. Moro reflex and intake and output are routine and not associated with IVH.

16. At a preconception counseling class, a client expresses concern and wonders how Healthy People 2030 will improve maternal-infant outcomes. Which response(s) by the nurse is appropriate? Select all that apply. A. Healthy People 2030 will reduce the rate of fetal and infant deaths. B. Healthy People 2030 will decrease the number of all infant deaths (within 1 year). C. Healthy People 2030 will decrease the number of neonatal deaths (within the first year). D. Healthy People 2030 will foster early and consistent prenatal care.

Answer: A, B, C, D Rationale: One of the leading health indicators as identified by Healthy People 2030 refers to decreasing the number of infant deaths. Acquired and congenital conditions account for a significant percentage of infant deaths. Specific objectives include reducing the rate of fetal deaths at 20 or more weeks of gestation though the nursing action of fostering early and consistent prenatal care; reducing the rate of all infant deaths (within 1 year) through the nursing actions of including education to place infants on their backs for naps and sleep to prevent sudden infant death syndrome (SIDS), avoiding exposing newborns to cigarette smoke, and ensuring that infants with birth defects receive health care needed in order to thrive; and reducing the occurrence of fetal alcohol syndrome (FAS) through the nursing actions or counseling girls and women to avoid alcohol use during pregnancy, and participating in programs for at-risk groups, including adolescents, about the effects of substance use, especially alcohol, during pregnancy.

17. A term neonate has been admitted to the observational newborn nursery with the diagnosis of being small for gestational age. Which factors would predispose the neonate to this diagnosis? Select all that apply. A. The mother had chronic placental abruption. B. At birth the placenta was noted to be decreased in weight. C. On assessment the placenta had areas of infarction. D. At birth the placenta was a shiny Schultz presentation. E. Placental talipes was present at birth.

Answer: A, B, C, D Rationale: Placental factors that can contribute to a small for gestational age infant include chronic placental abruption, infarction on surface of placenta, and a decreased placental weight. A shiny Schultz placenta is a normal description because the fetal side of the placenta comes out first, which is shiny. Placenta talipes does not exist.

27. A 20-hour-old neonate is suspected of having polycythemia. Which nursing intervention(s) will the nurse utilize to provide care for this neonate? Select all that apply. A. Obtain hemoglobin and hematocrit laboratory tests B. Provide early feedings to prevent hypoglycemia C. Maintain oxygen saturation parameters D. Monitor urinary output E. Insert a peripheral IV

Answer: A, B, C, D Rationale: Polycythemia in a neonate is defined as a hematocrit above 65% (0.65) and a hemoglobin level above 20 g/dl (200 g/l). The hematocrit and hemoglobin peak between 6 and 12 hours of life and then start to decrease. If these values do not decrease as expected, then hypoperfusion will occur and polycythemia will develop. In the beginning, the nurse may assess feeding difficulties, hypoglycemia, jitteriness and respiratory distress. As the condition worsens, a ruddy skin color could be seen, cyanosis could develop, the neonate could become lethargic and seizures could develop. Nursing care for this neonate requires obtaining hematocrit and hemoglobin laboratory tests at 2 hours, 12 hours and 24 hours. Feeding should be started to provide fluid, nutrition and prevent hypoglycemia. The oxygen saturation should be monitored. If the levels are below the established parameters from the health care provider, oxygen therapy will be needed. The urine output should be monitored continuously because polycythemia can cause real failure. A peripheral IV may or may not be needed. This would depend on the neonate's condition and if IV fluids would be required.

20. A neonate is born at 42 weeks' gestation weighing 4.4 kg (9 lb, 7 oz) with satisfactory Apgar scores. Two hours later birth the neonate's blood sugar indicates hypoglycemia. Which symptoms would the baby demonstrate? Select all that apply. A. poor sucking B. respiratory distress C. weak cry D. jitteriness E. blood glucose >40 mg/dl

Answer: A, B, C, D Rationale: Some of the common problems associated with newborns experiencing a variation in gestational age, such as a postterm newborn, are respiratory distress, jitteriness, feeble sucking, weak cry, and a blood glucose of 40 mg/dl.

ce: p. 846 29. A client expresses concerns that her grandmothers had complicated pregnancies. What principle(s) should the nurse discuss to allay the fears of the client? Select all that apply. A. "We work to ensure that birth of high-risk infants happens in settings where we are able to care for them." B. "We will work with you to identify prenatal risk factors early and take actions to reduce their impact." C. "We support those at risk of having a preterm births with the goal of delaying early births." D. "We work to ensure care for mothers and infants to reduce infant illnesses, disabilities, and death." E. "We allow families to grieve the loss of a newborn, should it occur."

Answer: A, B, C, D Rationale: The nurse will attempt to allay the client's fears by discussing the actions the facility enacts to promote a healthy brith and infant. This includes ensuring the birth of high-risk infants takes place in settings that have the technological capacity to care for them, identifying risk factors early and taking action to reduce their impact, working to delay the birth of those pregancies identified at risk of preterm birht, and promoting an overall reduction in infant illness, disability, and death to proper care of the mother and infant. Although allowing a family to greive in instances of infant death, discussing this factor with the client is likely to create more fear.

26. A nurse is reading a journal article about cesarean births and the indications for them. Place the indications for cesarean birth below in the proper sequence from most frequent to least frequent. All options must be used. A. Labor dystocia B. Abnormal fetal heart rate tracing C. Fetal malpresentation D. Multiple gestation E. Suspected macrosomia

Answer: A, B, C, D, E Rationale: The most common indications for primary cesarean births include, in order of frequency: labor dystocia as the labor does not progress, abnormal fetal heart rate tracing indicating fetal distress, fetal malpresentation making a difficult progression of labor, multiple gestation , and suspected macrosomia.

30. A pregnant client at 30 weeks' gestation calls the clinic because she thinks that she may be in labor. To determine if the client is experiencing labor, which question(s) would be appropriate for the nurse to ask? Select all that apply. A. "Are you feeling any pressure or heaviness in your pelvis?" B. "Are you having contractions that come and go, off and on?" C. "Have you noticed any fluid leaking from your vagina?" D. "Are you having problems with heartburn?" E. "Have you been having any nausea or vomiting?"

Answer: A, B, C, E Rationale: Frequently, women are unaware that uterine contractions, effacement, and dilation are occurring, thus making early intervention ineffective in arresting preterm labor and preventing the birth of a premature newborn. The nurse should ask the client about any signs/symptoms, being alert for subtle symptoms of preterm labor, which may include: a change or increase in vaginal discharge with mucous, water, or blood in it; pelvic pressure (pushing-down sensation); low dull backache; menstrual-like cramps; urinary tract infection symptoms; feeling of pelvic pressure or fullness; gastrointestinal upset like nausea, vomiting, and diarrhea; general sense of discomfort or unease; heaviness or aching in the thighs; uterine contractions with or without pain; more than six contractions per hour; intestinal cramping with or without diarrhea. Contractions also must be persistent, such that four contractions occur every 20 minutes or eight contractions occur in 1 hour. A report of heartburn is unrelated to preterm labor.

26. A neonate is admitted to the newborn observation nursery with the possible diagnosis of polycythemia. The nurse would be observing for which findings? Select all that apply. A. ruddy skin color B. respiratory distress C. cyanosis D. pink gums and tongue E. jitteriness

Answer: A, B, C, E Rationale: Observe for clinical signs of polycythemia (respiratory distress, cyanosis, jitteriness, jaundice, ruddy skin color, and lethargy) and monitor blood results.

21. A nurse is conducting an in-service program for a group of labor and birth unit nurses about cesarean birth. The group demonstrates understanding of the information when they identify which conditions as appropriate indications? Select all that apply. A. active genital herpes infection B. placenta previa C. previous cesarean birth D. prolonged labor E. fetal distress

Answer: A, B, C, E Rationale: The leading indications for cesarean birth are previous cesarean birth, breech presentation, dystocia, and fetal distress. Examples of specific indications include active genital herpes, fetal macrosomia, fetopelvic disproportion, prolapsed umbilical cord, placental abnormality (placenta previa or placental abruption), previous classic uterine incision or scar, gestational hypertension, diabetes, positive human immunodeficiency virus (HIV) status, and dystocia. Fetal indications include malpresentation (nonvertex presentation), congenital anomalies (fetal neural tube defects, hydrocephalus, abdominal wall defects), and fetal distress.

16. A 42-year-old woman is 26 weeks' pregnant. She lives at a shelter for female victims of intimate partner violence. Her blood pressure is 170/90 mm Hg, the fetal heart rate is 140 bpm, TORCH studies are positive, and she is bleeding vaginally. What findings put her at risk of giving birth to a small-for-gestational-age (SGA) infant? Select all that apply. A. the age of the client B. living in a shelter for victims of intimate partner violence C. vaginal bleeding D. fetal heart rate E. blood pressure F. positive test for TORCH

Answer: A, B, C, E, F Rationale: Some factors contributing to the birth of SGA newborns include maternal age of 20 or 35 years old, low socioeconomic status, and preeclampsia with increased blood pressure. The vaginal bleeding indicates placental problems, and she tests positive for sexually transmitted diseases by TORCH group infections.

18. A small-for-gestational age infant is admitted to the observational care unit with the nursing diagnosis of ineffective thermoregulation related to lack of fat stores as evidenced by persistent low temperatures. Which are appropriate nursing interventions? Select all that apply. A. Assess the axillary temperature every hour. B. Review maternal history. C. Assess environment for sources of heat loss. D. Bathe the neonate with warmer water. E. Minimize kangaroo care.F. Encourage skin-to-skin contact.

Answer: A, B, C, F Rationale: Proper care to promote thermoregulation include assessing the axillary temperature every hour, reviewing the maternal history to identify risk factors contributing to problem, assessing the environment for sources of heat loss, avoiding bathing and exposing newborn to prevent cold stress, and encouraging kangaroo care (mother or father holds preterm infant underneath clothing skin-to-skin and upright between breasts) to provide warmth. Question format: Multiple Select

24. A 30 weeks' gestation neonate born with low Apgar scores is in the neonatal intensive care unit with respiratory distress syndrome and underwent an exchange transfusion for anemia. Which factors place the neonate at risk for necrotizing enterocolitis? Select all that apply A. preterm birth B. respiratory distress syndrome C. low Apgar scores D. hyperthermia E. hyperglycemia F. exchange transfusion

Answer: A, B, C, F Rationale: The predisposing factors for the development of necrotizing enterocolitis include preterm labor, respiratory distress syndrome, exchange transfusion, and low birth weight. Low Apgar scores, hypothermia, and hypoglycemia are also risk factors.

14. While assessing a pregnant woman, the nurse suspects that the client may be at risk for hydramnios. Which information would the nurse use to support this suspicion? Select all that apply. A. history of diabetes B. reports of shortness of breath C. identifiable fetal parts on abdominal palpation D. difficulty obtaining fetal heart rate E. fundal height below that for expected gestational age

Answer: A, B, D Rationale: Factors such as maternal diabetes or multiple gestations place the woman at risk for hydramnios. In addition, there is a discrepancy between fundal height and gestational age, such that a rapid growth of the uterus is noted. Shortness of breath may result from overstretching of the uterus due to the increased amount of amniotic fluid. Often, fetal parts are difficult to palpate and fetal heart rate is difficult to obtain because of the excess fluid present.

14. A nurse is conducting a refresher program for a group of nurses working in the newborn nursery. After teaching the group about variations in newborn head size and appearance, the nurse determines that the teaching was successful when the group identifies which variation as normal? Select all that apply. A. cephalhematoma B. molding C. closed fontanels D. caput succedaneum E. posterior fontanel diameter 1.5 cm

Answer: A, B, D Rationale: Normal variations in newborn head size and appearance include cephalhematoma, molding, and caput succedaneum. Microcephaly, closed fontanels, or a posterior fontanel diameter greater than 1 cm are considered abnormal.

28. A 2-hour-old neonate born via caesarean birth has begun having a respiratory rate of 110 breaths/min and is in respiratory distress. What intervention(s) is a priority for the nurse to include in this neonates's care? Select all that apply. A. Keep the head in a "sniff" position B. Administer oxygen C. Insert an orogastric tube D. Ensure thermoregulation E. Obtain an arterial blood gas

Answer: A, B, D Rationale: This neonate is experiencing manifestations of transient tachypnea of the newborn (TTN). It occurs from delayed clearing of the lungs from fluid, and can be seen in neonates born via cesarean birth, because they have not had the experience of the compression on the thorax during vaginal delivery. This starts within the first 6 hours of life and can last up to 72 hours. The priority interventions for this neonate are oxygen, thermoregulation and minimal stimulation. Keeping the head in a neutral or "sniff " position allows for optimal airway. If the neonate becomes cold, then respiratory distress and or sepsis can develop. Minimal stimulation conserves the neonate's respiratory and heat requirements. The neonate may need placement of a peripheral IV for hydration and/or a feeding tube for formula or breast milk. The neonate should not be nipple fed until the respirations are under 60 breaths/min. A chest x-ray and an arterial blood gas may be needed also, but they would only be necessary if the neonate is in severe distress. The arterial blood gas results would show mild hypoxemia, a midly elevated CO2 level, and a normal pH.

19. A nurse is explaining to a group of nurses new to the labor and birth unit about methods used for cervical ripening. The group demonstrates understanding of the information when they identify which method as a mechanical one? A. herbal agents B. laminaria C. membrane stripping D. amniotomy

Answer: B Rationale: Laminaria is a hygroscopic dilator that is used as a mechanical method for cervical ripening. Herbal agents are a nonpharmacologic method. Membrane stripping and amniotomy are considered surgical methods.

16. A nurse is conducting a review course on tocolytic therapy for perinatal nurses. After teaching the group, the nurse determines that the teaching was successful when they identify which drugs as being used for tocolysis? Select all that apply. A. nifedipine B. magnesium sulfate C. dinoprostone D. misoprostol E. indomethacin

Answer: A, B, E Rationale: Medications most commonly used for tocolysis include magnesium sulfate (which reduces the muscle's ability to contract), indomethacin (a prostaglandin synthetase inhibitor), and nifedipine (a calcium channel blocker). These drugs are used "off label": this means they are effective for this purpose but have not been officially tested and developed for this purpose by the FDA. Dinoprostone and misoprostol are used to ripen the cervix.

29. A nurse is obtaining a medication history from a pregnant client with a history of systemic lupus erythematosus (SLE). Which medication(s) would the nurse expect the woman to report to be currently using? Select all that apply. A. Ibuprofen B. Hydroxychloroquine C. Methotrexate D. Leflunomide E. Prednisone

Answer: A, B, E Rationale: Treatment of SLE in pregnancy is generally limited to NSAIDs like ibuprofen, prednisone, and an antimalarial agent, hydroxychloroquine. Methotrexate and leflunomide are used to treat rheumatoid arthritis but are contraindicated for use in pregnancy because of the potential for fetal toxicity.

22. A nurse is providing a refresher class for a group of postpartum nurses. The nurse reviews the risk factors associated with postpartum hemorrhage. The group demonstrates understanding of the information when they identify which risk factors associated with uterine tone? Select all that apply. A. rapid labor B. retained blood clots C. hydramnios D. operative birth E. fetal malpostion

Answer: A, C Rationale: Risk factors associated with uterine tone include hydramnios, rapid or prolonged labor, oxytocin use, maternal fever, or prolonged rupture of membranes. Retained blood clots are a risk factor associated with tissue retained in the uterus. Fetal malposition and operative birth are risk factors associated with trauma of the genital tract.

20. The nurse notifies the obstetrical team immediately because the nurse suspects that the pregnant woman may be exhibiting signs and symptoms of amniotic fluid embolism. When reporting this suspicion, which finding(s) would the nurse include in the report? Select all that apply. A. significant difficulty breathing B. hypertension C. tachycardia D. pulmonary edema E. bleeding with bruising

Answer: A, C, D, E Rationale: Anaphylactoid syndrome of pregnancy (ASP), also known as amniotic fluid embolism, is an unforeseeable, life-threatening complication of childbirth. The etiology of ASP remains an enigmatic, devastating obstetric condition associated with significant maternal and newborn morbidity and mortality. It is a rare and often fatal event characterized by the sudden onset of hypotension, cardiopulmonary collapse, hypoxia, and coagulopathy. ASP should be suspected in any pregnant women with an acute onset of dyspnea, hypotension, and DIC. By knowing how to intervene, the nurse can promote a better chance of survival for both the mother and her newborn.

23. A nurse is preparing a class on newborn adaptations for a group of soon-to-be parents. When describing the change from fetal to newborn circulation, which information would the nurse include? Select all that apply. A. Decrease in right atrial pressure leads to closure of the foramen ovale. B. Increase in oxygen levels leads to a decrease in systemic vascular resistance. C. Onset of respirations leads to a decrease in pulmonary vascular resistance. D. Increase in pressure in the left atrium results from increases in pulmonary blood flow. E. Closure of the ductus venosus eventually forces closure of the ductus arteriosus.

Answer: A, C, D, E Rationale: When the umbilical cord is clamped, the first breath is taken, and the lungs begin to function. As a result, systemic vascular resistance increases and blood return to the heart via the inferior vena cava decreases. Concurrently with these changes, there is a rapid decrease in pulmonary vascular resistance and an increase in pulmonary blood flow (Boxwell, 2010). The foramen ovale functionally closes with a decrease in pulmonary vascular resistance, which leads to a decrease in right-sided heart pressures. An increase in systemic pressure, after clamping of the cord, leads to an increase in left-sided heart pressures. Ductus arteriosus, ductus venosus, and umbilical vessels that were vital during fetal life are no longer needed.

29. A 32-year-old black woman in her second trimester has come to the clinic for an evaluation. While interviewing the client, she reports a history of fibroids and urinary tract infection. The client states, "I know smoking is bad and I have tried to stop, but it is impossible. I have cut down quite a bit though, and I do not drink alcohol." Complete blood count results reveal a low red blood cell count, low hemoglobin, and low hematocrit. When planning this client's care, which factor(s) would the nurse identify as increasing the client's risk for preterm labor? Select all that apply. A. African heritage B. Maternal age C. History of fibroids D. Cigarette smoking E. History of urinary tract infections F. Complete blood count results

Answer: A, C, D, E, F Rationale: For this client, risk factors associated with preterm labor and birth would include African heritage, cigarette smoking, uterine abnormalities, such as fibroids, urinary tract infection, and possible anemia based on her complete blood count results. Maternal age extremes (younger than 16 years and older than 35 years) are also a risk factor but do not apply to this client.

25. A nurse is conducting a presentation for a group of pregnant women about measures to prevent toxoplasmosis. The nurse determines that additional teaching is needed when the group identifies which measure as preventive? A. washing raw fruits and vegetables before eating them B. cooking all meat to an internal temperature of 125° F (52° C) C. wearing gardening gloves when working in the soil D. avoiding contact with a cat's litter box

Answer: B Rationale: Meats should be cooked to an internal temperature of 160° F (71° C). Other measures to prevent toxoplasmosis include peeling or thoroughly washing all raw fruits and vegetables before eating them, wearing gardening gloves when in contact with outdoor soil, and avoiding the emptying or cleaning of a cat's litter box.

12. A 22-year-old woman experiencing homelessness arrives at a walk-in clinic seeking pregnancy confirmation. The nurse notes on assessment her uterus suggests 12 weeks' gestation, a blood pressure of 110/70 mm Hg, and a BMI of 17.5. The client admits to using cocaine a few times. The client has been pregnant before and indicates she "loses them early." What characteristic(s) place the client in the high-risk pregnancy category? Select all that apply. A. BMI 17.5 B. blood pressure 110/70 mm Hg C. prenatal history D. homelessness E. age F. prenatal care

Answer: A, C, D, F Rationale: The key to identifying a newborn with special needs related to birthweight or gestational age variation is an awareness of the factors that could place a newborn at risk. These factors are similar to those that would suggest a high-risk pregnancy and include maternal nutrition (malnutrition or overweight), substandard living conditions or low socioeconomic status, maternal age of less than 20 or more than 35 years, lack of prenatal care, and history of previous preterm birth.

20. A newborn is experiencing cold stress. Which findings would the nurse expect to assess? Select all that apply. A. respiratory distress B. decreased oxygen needs C. hypoglycemia D. metabolic alkalosis E. jaundice

Answer: A, C, E Rationale: Cold stress in the newborn can lead to the following problems if not reversed: depleted brown fat stores, increased oxygen needs, respiratory distress, increased glucose consumption leading to hypoglycemia, metabolic acidosis, jaundice, hypoxia, and decreased surfactant production.

26. A nurse is teaching a woman with mild preeclampsia about important areas that she needs to monitor at home. The nurse determines that the teaching was successful based on which statements by the woman? Select all that apply. A. "I should check my blood pressure twice a day." B. "I will weigh myself once a week." C. "I should complete a fetal kick count each day." D. "I will check my urine for protein four times a day." E. "I'll call my health care provider if I have burning when I urinate."

Answer: A, C, E Rationale: The client should take her blood pressure twice daily, check and record weight daily, perform urine dipstick checks for protein twice daily, record the number of fetal kicks daily, and notify her health care provider if she experiences burning on urination.

16. A nurse has been invited to speak at a local high school about adolescent pregnancy. When developing the presentation, the nurse would incorporate information related to which aspects? Select all that apply. A. peer pressure to become sexually active B. rise in teen birth rates over the years. C. Asian Americans as having the highest teen birth rate D. loss of self-esteem as a major impact E. about half occurring within a year of first sexual intercourse

Answer: A, D

21. Assessment of a postpartum woman experiencing postpartum hemorrhage reveals mild shock. Which finding would the nurse expect to assess? Select all that apply. A. diaphoresis B. tachycardia C. oliguria D. cool extremities E. confusion

Answer: A, D Rationale: Signs and symptoms of mild shock include diaphoresis, increased capillary refill, cool extremities, and maternal anxiety. Tachycardia and oliguria suggest moderate shock. Confusion suggests severe shock.

19. A nurse suspects that a pregnant client may be experiencing a placental abruption based on assessment of which finding? Select all that apply. A. dark red vaginal bleeding B. insidious onset C. absence of pain D. rigid uterus E. absent fetal heart tones

Answer: A, D, E Rationale: Assessment findings associated with a placental abruption include a sudden onset with concealed or visible dark red bleeding, constant pain or uterine tenderness on palpation, firm to rigid uterine tone, and fetal distress or absent fetal heart tones.

30. A new parent is talking with the nurse about feeding the newborn. The parent has chosen to use formula. The parent asks, "How can I make sure that my baby is getting what is needed?" Which response(s) by the nurse would be appropriate? Select all that apply. A. "Make sure to use an iron-fortified formula until your baby is about 1 year old." B. "Start giving your baby fluoride supplements now so your baby develops strong teeth." C. "Since you are not breastfeeding, your baby needs a baby multivitamin each day." D. "Your baby gets enough fluid with formula, so you do not need to give extra water." E. "It is important to give your baby vitamin D each day."

Answer: A, D, E Rationale: Fluid requirements for the newborn and infant range from 100 to 150 mL/kg daily. This requirement can be met through breastfeeding or bottle feeding. Additional water supplementation is not necessary. Adequate carbohydrates, fats, protein, and vitamins are achieved through consumption of breast milk or formula. Iron-fortified formula is recommended for all infants who are not breastfed from birth to 1 year of age. The breastfed infant draws on iron reserves for the first 6 months and then needs iron-rich foods or supplementation added at 6 months of age. All infants (breastfed and bottle fed) should receive a daily supplement of 400 International Units of vitamin D starting within the first few days of life to prevent rickets and vitamin D deficiency. It is also recommended that fluoride supplementation be given to infants not receiving fluoridated water after the age of 6 months.

17. A home health care nurse is assessing a postpartum woman who was discharged 2 days ago. The woman tells the nurse that she has a low-grade fever and feels "lousy." Which finding would lead the nurse to suspect endometritis? Select all that apply. A. lower abdominal tenderness B. urgency C. flank pain D. breast tenderness E. anorexia

Answer: A, E Rationale: Manifestations of endometritis include lower abdominal tenderness or pain on one or both sides, elevated temperature, foul-smelling lochia, anorexia, nausea, fatigue and lethargy, leukocytosis, and elevated sedimentation rate. Urgency and flank pain would suggest a urinary tract infection. Breast tenderness may be related to engorgement or suggest mastitis.

30. A client at 33 weeks' gestation comes to the emergency department with vaginal bleeding. Assessment reveals the following: Onset of slight vaginal bleeding at 29 weeks with spontaneous cessation Recent onset of bright red vaginal bleeding, more than with previous episode No uterine contractions at present Fetal heart rate within normal range Uterus soft and nontender Based on the assessment findings, which condition would the nurse likely suspect? A. Placental abruption B. Placenta previa C. Ruptured ectopic pregnancy D. Polyhydramnios

Answer: B

10. A postpartum client is prescribed medication therapy as part of the treatment plan for postpartum hemorrhage. Which medication would the nurse expect to administer in this situation? A. Magnesium sulfate B. methylergonovine C. Indomethacin D. nifedipine

Answer: B Rationale: Methylergonovine, along with oxytocin and carboprost are drugs used to manage postpartum hemorrhage. Magnesium sulfate, indomethecin, and nifedipine are used to control preterm labor.

10. Assessment of a newborn reveals uneven gluteal (buttocks) skin creases and a "clunk" when the Ortolani maneuver is performed. What would the nurse suspect? A. slipping of the periosteal joint B. developmental hip dysplasia C. normal newborn variation D. overriding of the pelvic bone

Answer: B Rationale: A "clunk" indicates the femoral head hitting the acetabulum as the head reenters the area. This, along with uneven gluteal creases, suggests developmental hip dysplasia. These findings are not a normal variation and are not associated with slipping of the periosteal joint or overriding of the pelvic bone.

8. A woman receives magnesium sulfate as treatment for preterm labor. The nurse assess and maintains the infusion at the prescribed rate based on which finding? A. Respiratory rate-16 breaths/minute B. Decreased fetal heart rate variability C. Urine output 22 mL/hour D. Absent deep tendon reflexes

Answer: B Rationale: A respiratory rate of 16 breaths per minute is appropriate and within acceptable parameters to continue the infusion. When administering magnesium sulfate, the nurse would immediately report decreaed fetal heart rate variability, a urine output less than 30 mL/hour, and decreased or absent deep tendon reflexes.

17. The nurse observes the stool of a newborn who has begun to breastfeed. Which finding would the nurse expect? A. greenish black, tarry stool B. yellowish-brown, seedy stool C. yellow-gold, stringy stool D. yellowish-green, pasty stool

Answer: B Rationale: After feedings are initiated, a transitional stool develops, which is greenish brown to yellowish brown, thinner in consistency, and seedy in appearance. Meconium stool is greenish black and tarry. The last development in the stool pattern is the milk stool. Milk stools of the breastfed newborn are yellow-gold, loose, and stringy to pasty in consistency, and typically sour- smelling. The milk stools of the formula-fed newborn vary depending on the type of formula ingested. They may be yellow, yellow-green, or greenish and loose, pasty, or formed in consistency, and they have an unpleasant odor.

4. A nurse is developing a program to help reduce the risk of late postpartum hemorrhage in clients in the labor and birth unit. Which measure would the nurse emphasize as part of this program? A. administering broad-spectrum antibiotics B. inspecting the placenta after delivery for intactness C. manually removing the placenta at birth D. applying pressure to the umbilical cord to remove the placenta

Answer: B Rationale: After the placenta is expelled, a thorough inspection is necessary to confirm its intactness because tears or fragments left inside may indicate an accessory lobe or placenta accreta. These can lead to profuse hemorrhage because the uterus is unable to contract fully. Administering antibiotics would be appropriate for preventing infection, not postpartum hemorrhage. Manual removal of the placenta or excessive traction on the umbilical cord can lead to uterine inversion, which in turn would result in hemorrhage.

1. A nurse is explaining to a group of new parents about the changes that occur in the neonate to sustain extrauterine life, describing the cardiac and respiratory systems as undergoing the most changes. Which information would the nurse integrate into the explanation to support this description? A. The cardiac murmur heard at birth disappears by 48 hours of age. B. Pulmonary vascular resistance (PVR) is decreased as lungs begin to function. C. Heart rate remains elevated after the first few moments of birth. D. Breath sounds will have rhonchi for at least the first day of life as fluid is absorbed.

Answer: B Rationale: Although all the body systems of the newborn undergo changes, respiratory gas exchange along with circulatory modifications must occur immediately to sustain extrauterine life. With the first breath, PVR decreases, and the heart rate initially increases but then decreases to 120 to 130 bpm after a few minutes. The ductal murmur will go away in 80+% of infants by 48 hours. Rhonchi caused by retained amniotic fluid is an abnormal finding and would not be expected.

25. A woman is receiving magnesium sulfate as part of her treatment for severe preeclampsia. The nurse is monitoring the woman's serum magnesium levels. The nurse determines that the drug is at a therapeutic level based on which result? A. 3.3 mEq/L B. 6.1 mEq/L C. 8.4 mEq/L D. 10.8 mEq/L

Answer: B Rationale: Although exact levels may vary among agencies, serum magnesium levels ranging from 4 to 7 mEq/L are considered therapeutic, whereas levels more than 8 mEq/dL are generally considered toxic.

11. A newborn has an Apgar score of 6 at 5 minutes. Which action would be the priority? A. initiating IV fluid therapy B. beginning resuscitative measures C. promoting kangaroo care D. obtaining a blood culture

Answer: B Rationale: An Apgar score below 7 at 1 or 5 minutes indicates the need for resuscitation. Intravenous fluid therapy and blood cultures may be done once resuscitation is started. Kangaroo care would be appropriate once the newborn is stable.

24. A woman with gestational hypertension develops eclampsia and experiences a seizure. Which intervention would the nurse identify as the PRIORITY? A. fluid replacement B. oxygenation C. control of hypertension D. birth of the fetus

Answer: B Rationale: As with any seizure, the priority is to clear the airway and maintain adequate oxygenation both to the mother and the fetus. Fluids and control of hypertension are addressed once the airway and oxygenation are maintained. Delivery of fetus is determined once the seizures are controlled and the woman is stable.

27. A client comes to the emergency department with moderate vaginal bleeding. She says, "I have had to change my pad about every 2 hours and it looks like I may have passed some tissue and clots." The woman reports that she is 9 weeks' pregnant. Further assessment reveals the following: Cervical dilation Strong abdominal cramping Low human chorionic gonadotropin (hCG) levels Ultrasound positive for products of conception The nurse suspects that the woman is experiencing which type of spontaneous abortion? A. Threatened B. Inevitable C. Incomplete D. Complete

Answer: B Rationale: Based on the assessment findings, the woman is likely experiencing an inevitable abortion characterized by vaginal bleeding, rupture of membranes, cervical dilation, strong abdominal cramping, possible passage of products of conception, and ultrasound and hCG levels indicating pregnancy loss. A threatened abortion is characterized by slight vaginal bleeding, no cervical dilation or cange in cervical consistency, mild abdominal cramping, close cervical os, and no passage of fetal tissue. An incomplete abortion is characterized by intense abdominal cramping, heavy vaginal bleeding and cervical dilation with passage of some products of conception. A complete abortion is characterized by a history of vaginal bleeding and abdominal pain along with passage of tissue and subsequent decrease in pain and decrease in bleeding.

10. A woman with preterm labor is receiving magnesium sulfate. Which finding would require the nurse to intervene immediately?A. respiratory rate of 16 breaths per minute B. 1+ deep tendon reflexes C. urine output of 45 mL/hour D. alert level of consciousness

Answer: B Rationale: Diminished deep tendon reflexes (1+) suggest magnesium toxicity, which requires immediate intervention. Additional signs of magnesium toxicity include a respiratory rate less than 12 breaths/minute, urine output less than 30 mL/hour, and a decreased level of consciousness.

18. A nurse is describing the risks associated with post-term pregnancies as part of an inservice presentation. The nurse determines thatmore teaching is needed when the group identifies which factor as an underlying reason for problems in the fetus?A. aging of the placenta B. increased amniotic fluid volume C. meconium aspiration D. cord compression

Answer: B Rationale: Fetal risks associated with a post-term pregnancy include macrosomia, shoulder dystocia, brachial plexus injuries, low Apgar scores, postmaturity syndrome (loss of subcutaneous fat and muscle and meconium staining), and cephalopelvic disproportion. As the placenta ages, its perfusion decreases and it becomes less efficient at delivering oxygen and nutrients to the fetus. Amniotic fluid volume also begins to decline after 38 weeks' gestation, possibly leading to oligohydramnios, subsequently resulting in fetal hypoxia and an increased risk of cord compression because the cushioning effect offered by adequate fluid is no longer present. Hypoxia and oligohydramnios predispose the fetus to aspiration of meconium, which is released by the fetus in response to a hypoxic insult (Norwitz, 2019). All of these issues can compromise fetal well-being and lead to fetal distress.

29. A client who has experienced an incomplete abortion is prescribed mifepristone to assist in removing the retained products of conception. Which medication would the nurse expect to adminster if prescribed before administering mifepristone? A. Opioid analgesic for relief of cramping B. Antiemetic to minimize nausea C. VItamin K to reduce bleeding D. Diuretic to promote fluid loss

Answer: B Rationale: For the client receiving mifepristone, the nurse would anticipate administering an antiemetic beforehand to reduce nausea and vomiting. Acetaminophen would be useful for pain relief, not an opioid. VItamin K or a diuretic would not be appropriate when administering mifepristone. Vitamin K would be used to counteract bleeding such as that associated with heparin administration. A diuretic would be appropriate to promote fluid excretion with fluid overload.

6. The nurse prepares to administer a gavage feeding for a newborn with transient tachypnea based on the understanding that this type of feeding is necessary because: A. lactase enzymatic activity is not adequate. B. oxygen demands need to be reduced. C. renal solute lead must be considered. D. hyperbilirubinemia is likely to develop.

Answer: B Rationale: For the newborn with transient tachypnea, the newborn's respiratory rate is high, increasing the oxygen demand. Thus, measures are initiated to reduce this demand. Gavage feedings are one way to do so. With transient tachypnea, enzyme activity and kidney function are not affected. This condition typically resolves within 72 hours. The risk for hyperbilirubinemia is not increased.

7. When preparing a schedule of follow-up visits for a pregnant woman with chronic hypertension, which schedule would be most appropriate? A. monthly visits until 32 weeks, then bi-monthly visits B. bi-monthly visits until 28 weeks, then weekly visits C. monthly visits until 20 weeks, then bi-monthly visits D. bi-monthly visits until 36 weeks, then weekly visits

Answer: B Rationale: For the woman with chronic hypertension, antepartum visits typically occur every 2 weeks until 28 weeks' gestation and then weekly to allow for frequent maternal and fetal surveillance.

30. A nurse is teaching a new mother about her newborn's immune status. The nurse determines that the teaching was successful when the mother states which immunoglobulin as having crossed the placenta? A. IgA B. IgG C. IgM D. IgE

Answer: B Rationale: IgG is the major immunoglobulin and the most abundant, making up about 80% of all circulating antibodies. It is found in serum and interstitial fluid. It is the only class able to cross the placenta, with active placental transfer beginning at approximately 20 to 22 weeks' gestation. No other immunoglobulin crosses the placenta.

12. A nurse is conducting an in-service presentation to a group of perinatal nurses about sexually transmitted infections and their effect on pregnancy. The nurse determines that the teaching was successful when the group identifies which infection as being responsible for ophthalmia neonatorum? A. syphilis B. gonorrhea C. chlamydia D. HPV

Answer: B Rationale: Infection with gonorrhea during pregnancy can cause ophthalmia neonatorum in the newborn from birth through an infected birth canal. Infection with syphilis can cause congenital syphilis in the neonate. Infection with chlamydia can lead to conjunctivitis or pneumonia in the newborn. Exposure to HPV during birth is associated with laryngeal papillomas.

5. The nurse prepares to assess a newborn who is considered to be large-for-gestational-age (LGA). Which characteristic would the nurse correlate with this gestational age variation? A. strong, brisk motor skills B. difficulty in arousing to a quiet alert state C. birthweight of 7 lb, 14 oz (3,572 g) D. wasted appearance of extremities

Answer: B Rationale: LGA newborns typically are more difficult to arouse to a quiet alert state. They have poor motor skills, have a large body that appears plump and full-sized, and usually weigh more than 8 lb, 13 oz (3,997 g) at term.

10. The nurse is assessing a newborn of a woman who is suspected of abusing alcohol. Which newborn finding would provide additional evidence to support this suspicion? A. wide, large eyes B. thin upper lip C. protruding jaw D. elongated nose

Answer: B Rationale: Newborn characteristics suggesting fetal alcohol spectrum disorder include thin upper lip, small head circumference, small eyes, receding jaw, and short nose. Other features include a low nasal bridge, short palpebral fissures, flat midface, epicanthal folds, and minor ear abnormalities.

22. A nurse is reviewing the laboratory test results of a newborn. Which result would the nurse identify as a cause for concern? A. hemoglobin 19 g/dL B. platelets 75,000/μL C. white blood cells 20,000/mm3 D. hematocrit 52%

Answer: B Rationale: Normal newborn platelets range from 150,00 to 350,000/μL. Normal hemoglobin ranges from 17 to 23g/dL, and normal hematocrit ranges from 46% to 68%. Normal white blood cell count ranges from 10,000 to 30,000/mm3.

7. Which information would the nurse include when teaching a new mother about the difference between pathologic and physiologic jaundice? A. Physiologic jaundice results in kernicterus. B. Pathologic jaundice appears within 24 hours after birth. C. Both are treated with exchange transfusions of maternal O- blood. D. Physiologic jaundice requires transfer to the NICU

Answer: B Rationale: Pathologic jaundice appears within 24 hours after birth whereas physiologic jaundice commonly appears around the third or fourth days of life. Kernicterus is more commonly associated with pathologic jaundice. An exchange transfusion is used only if the total serum bilirubin level remains elevated after intensive phototherapy. With this procedure, the newborn's blood is removed and replaced with nonhemolyzed red blood cells from a donor. Physiologic jaundice often is treated at home.

10. After teaching the parents of a newborn with periventricular hemorrhage about the disorder and treatment, which statement by the parents indicates that the teaching was successful? A. "We'll make sure to cover both of his eyes to protect them." B. "Our newborn could develop a learning disability later on." C. "Once the bleeding ceases, there won't be any more worries." D. "We need to get family members to donate blood for transfusion."

Answer: B Rationale: Periventricular hemorrhage has long-term sequelae such as seizures, hydrocephalus, periventricular leukomalacia, cerebral palsy, learning disabilities, vision or hearing deficits, and intellectual disability. Covering the eyes is more appropriate for the newborn receiving phototherapy. The bleeding in the brain can lead to serious long-term effects. Blood transfusions are not used to treat periventricular hemorrhage.

20. A nurse is teaching new parents about bathing their newborn. The nurse determines that the teaching was successful when the parents make which statement? A. "We can put a tiny bit of lotion on his skin, and then rub it in gently." B. "We should avoid using any kind of baby powder." C. "We need to bathe him at least four to five times a week." D. "We should clean his eyes after washing his face and hair."

Answer: B Rationale: Powders should not be used, because they can be inhaled, causing respiratory distress. If the parents want to use oils and lotions, have them apply a small amount onto their hand first, away from the newborn; this warms the lotion. Then the parents should apply the lotion or oil sparingly. Parents need to be instructed that a bath two or three times weekly is sufficient for the first year because too frequent bathing may dry the skin. The eyes are cleaned first and only with plain water; then the rest of the face is cleaned with plain water.

3. Which assessment finding will alert the nurse to be on the lookout for possible placental abruption during labor? A. macrosomia B. gestational hypertension C. gestational diabetes D. low parity

Answer: B Rationale: Risk factors for placental abruption include preeclampsia, gestational hypertension, seizure activity, uterine rupture, trauma, smoking, cocaine use, coagulation defects, previous history of abruption, intimate partner violence, and placental pathology. Macrosomia, gestational diabetes, and low parity are not considered risk factors.

25. A newborn is scheduled to undergo a screening test for phenylketonuria (PKU). The nurse prepares to obtain the blood sample from the newborn's: A. finger. B. heel. C. scalp vein. D. umbilical vein.

Answer: B Rationale: Screening tests for genetic and inborn errors of metabolism require a few drops of blood taken from the newborn's heel. The finger, scalp vein, or umbilical vein are inappropriate sites for the blood sample.

18. A postpartum client comes to the clinic for her routine 6-week visit. The nurse assesses the client and suspects that she is experiencing subinvolution based on which finding?' A. nonpalpable fundus B. moderate lochia serosa C. bruising on arms and legs D. fever

Answer: B Rationale: Subinvolution is usually identified at the woman's postpartum examination 4 to 6 weeks after birth. The clinical picture includes a postpartum fundal height that is higher than expected, with a boggy uterus; the lochia fails to change colors from red to serosa to alba within a few weeks. Normally, at 4 to 6 weeks, lochia alba or no lochia would be present and the fundus would not be palpable. Thus evidence of lochia serosa suggests subinvolution. Bruising would suggest a coagulopathy. Fever would suggest an infection.

31. A nurse is assessing a client who gave birth vaginally about 4 hours ago. The client tells the nurse that she changed her perineal pad about an hour ago. On inspection, the nurse notes that the pad is now saturated. The uterus is firm and approximately at the level of the umbilicus. Further inspection of the perineum reveals an area, bluish in color and bulging just under the skin surface. Which action would the nurse do next? A. Apply warm soaks to the area. B. Notify the health care provider. C. Massage the uterine fundus. D. Encourage the client to void.

Answer: B Rationale: The client is experiencing postpartum hemorrhage secondary to a perineal hematoma. The nurse needs to notify the health care provider about these findings to prevent further hemorrhage. Applying warm soaks to the area would do nothing to control the bleeding. With a perineal hematoma, the uterus is firm, so massaging the uterus or encouraging the client to void would not be appropriate

21. A nurse is reviewing an article about preterm prelabor rupture of membranes. Which factors would the nurse expect to find placing a woman at high risk for this condition? Select all that apply. A. high body mass index B. urinary tract infection C. low socioeconomic status D. single gestations E. smoking

Answer: B, C, E Rationale: High-risk factors associated with prelabor rupture of membranes (PROM) include low socioeconomic status, multiple gestation, low body mass index, tobacco use, preterm labor history, placenta previa, abruptio placenta, urinary tract infection, vaginal bleeding at any time in pregnancy, cerclage, and amniocentesis.

22. A nurse is teaching a postpartum client and her partner about caring for their newborn's umbilical cord site. Which statement by the parents indicates a need for additional teaching? A. "We can put him in the tub to bathe him once the cord falls off and is healed." B. "The cord stump should change from brown to yellow." C. "Exposing the stump to the air helps it to dry." D. "We need to call the primary care provider if we notice a funny odor."

Answer: B Rationale: The cord stump should change color from yellow to brown or black. Therefore the parents need additional teaching if they state the color changes from brown to yellow. Tub baths are avoided until the cord has fallen off and the area is healed. Exposing the stump to the air helps it to dry. The parents should notify their primary care provider if there is any bleeding, redness, drainage, or foul odor from the cord stump.

14. A nurse is reviewing a journal article on the causes of postpartum hemorrhage. Which condition would the nurse most likely find as the most common cause? A. labor augmentation B. uterine atony C. cervical or vaginal lacerations D. uterine inversion

Answer: B Rationale: The most common cause of postpartum hemorrhage is uterine atony, failure of the uterus to contract and retract after birth. The uterus must remain contracted after birth to control bleeding from the placental site. Labor augmentation is a risk factor for postpartum hemorrhage. Lacerations of the birth canal and uterine inversion may cause postpartum hemorrhage, but these are not the most common cause.

16. A group of nurses are reviewing information about mastitis and its causes in an effort to develop a teaching program on prevention for postpartum women. The nurses demonstrate understanding of the information when they focus the teaching on ways to minimize risk of exposure to which organism? A. E. coli B. S. aureus C. Proteus D. Klebsiella

Answer: B Rationale: The most common infectious organism that causes mastitis is S. aureus, which comes from the breast-feeding infant's mouth or throat. E. coli is another, less common cause. E. coli, Proteus, and Klebsiella are common causes of urinary tract infections.

12. The nurse is teaching a group of parents about the similarities and differences between newborn skin and adult skin. Which statement by the group indicates that additional teaching is needed? A. "The newborn's skin and that of an adult are similar in thickness." B. "The newborn's sweat glands function fully, just like those of an adult." C. "Skin development in the newborn is not complete at birth." D. "The newborn has fewer fibrils connecting the dermis and epidermis."

Answer: B Rationale: The newborn has sweat glands, like an adult, but full adult functioning is not present until the second or third year of life. The newborn and adult epidermis is similar in thickness and lipid composition, but skin development is not complete at birth. Fewer fibrils connect the dermis and epidermis in the newborn when compared with the adult.

16. Assessment of a newborn's head circumference reveals that it is 34 cm. The nurse would suspect that this newborn's chest circumference would be: A. 30 cm. B. 32 cm. C. 34 cm. D. 36 cm.

Answer: B Rationale: The newborn's chest should be round, symmetric, and 2 to 3 cm smaller than the head circumference. Therefore, this newborn's chest circumference of 31 to 32 cm would be normal.

23. A nurse is massaging a postpartum client's fundus and places the nondominant hand on the area above the symphysis pubis based on the understanding that this action: A. determines that the procedure is effective. B. helps support the lower uterine segment. C. aids in expressing accumulated clots. D. prevents uterine muscle fatigue.

Answer: B Rationale: The nurse places the nondominant hand on the area above the symphysis pubis to help support the lower uterine segment. The hand, usually the dominant hand that is placed on the fundus, helps to determine uterine firmness (and thus the effectiveness of the massage). Applying gentle downward pressure on the fundus helps to express clots. Overmassaging the uterus leads to muscle fatigue.

30. A late preterm newborn is being prepared for discharge to home after being in the neonatal intensive care unit for 4 days. The nurse instructs the parents about the care of their newborn and emphasizes warning signs that should be reported to the pediatrician immediately. The nurse determines that additional teaching is needed based on which parental statement? A. "We will call 911 if we start to see that our newborn's lips or skin are looking bluish." B. "If our newborn's skin turns yellow, it is from the treatments and our newborn is okay." C. "If our newborn does not have a wet diaper in 12 hours, we will call our pediatrician." D. "We will let the pediatrician know if our newborn's temperature goes above 100.4°F (38°C)."

Answer: B Rationale: The parents of a preterm newborn need teaching about when to notify their pediatrician or nurse practitioner. These include: displaying a yellow color to the skin (jaundice); having difficulty breathing or turning blue (call for emergency services in this case); having a temperature below 97°F (36.1°C) or above 100.4°F (38°C); and failing to void for 12 hours.

12. When describing the stages of labor to a pregnant woman, which of the following would the nurse identify as the major change occurring during the first stage? A. Regular contractions B. Cervical dilation C. Fetal movement through the birth canal D. Placental separation

Answer: B Rationale: The primary change occurring during the first stage of labor is progressive cervical dilation. Contractions occur during the first and second stages of labor. Fetal movement through the birth canal is the major change during the second stage of labor. Placental separation occurs during the third stage of labor.

24. A nurse is describing the advantages and disadvantages of circumcision to a group of expectant parents. Which statement by the parents indicates effective teaching? A. "Sexually transmitted infections are more common in circumcised males." B. "The rate of penile cancer is less for circumcised males." C. "Urinary tract infections are more easily treated in circumcised males." D. "Circumcision is a risk factor for acquiring HIV infection."

Answer: B Rationale: The risk for penile cancer appears to be slightly lower for males who are circumcised. However, penile cancer is rare and other risk factors such as genital warts and HPV infection seem to play a larger role. Sexually transmitted infections are less common in circumcised males, but the risk is believed to be related more to behavioral factors than circumcision status. Circumcised males have a 50% lower risk of acquiring HIV infection. Urinary tract infections are slightly less common in circumcised boys. However, rates are low in both circumcised and uncircumcised boys and are easily treated without long-term sequelae.

24. A woman with a history of systemic lupus erythematosus comes to the clinic for evaluation. The woman tells the nurse that she and her partner would like to have a baby but that they are afraid her lupus will be a problem. Which response would be most appropriate for the nurse to make? A. "It's probably not a good idea for you to get pregnant since you have lupus." B. "Be sure that your lupus is stable or in remission for 6 months before getting pregnant." C. "Your lupus will not have any effect on your pregnancy whatsoever." D. "If you get pregnant, we'll have to add quite a few medications to your normal treatment plan.

Answer: B Rationale: The time at which the nurse comes in contact with the woman in her childbearing life cycle will determine the focus of the assessment. If the woman is considering pregnancy, it is recommended that she postpone conception until the disease has been stable or in remission for 6 months. Active disease at time of conception and history of renal disease increase the likelihood of a poor pregnancy outcome (Cunningham et al., 2018). In particular, if pregnancy is planned during periods of inactive or stable disease, the result is often giving birth to healthy full-term babies without increased risks of pregnancy complications. Nonetheless, pregnancies with most autoimmune diseases are still classified as high risk because of the potential for major complications. Preconception counseling should include the medical and obstetric risks of spontaneous abortion, stillbirth, fetal death, fetal growth restriction, preeclampsia, preterm labor, and neonatal death and the need for more frequent visits for monitoring the condition. Treatment of SLE in pregnancy is generally limited to NSAIDs (e.g., ibuprofen), prednisone, and an antimalarial agent, hydroxychloroquine. During pregnancy in the woman with SLE, the goal is to keep drug therapy to a minimum.

2. A primigravida whose labor was initially progressing normally is now experiencing a decrease in the frequency and intensity of her contractions. The nurse would assess the woman for which condition? A. a low-lying placenta B. fetopelvic disproportion C. contraction ring D. uterine bleeding

Answer: B Rationale: The woman is experiencing dystocia most likely due to hypotonic uterine dysfunction and fetopelvic disproportion associated with a large fetus. A low-lying placenta, contraction ring, or uterine bleeding would not be associated with a change in labor pattern.

24. A nurse is preparing a presentation about ways to minimize heat loss in the newborn. Which measure would the nurse include to prevent heat loss through convection? A. placing a cap on a newborn's head B. working inside an isolette as much as possible. C. placing the newborn skin-to-skin with the mother D. using a radiant warmer to transport a newborn

Answer: B Rationale: To prevent heat loss by convection, the nurse would keep the newborn out of direct cool drafts (open doors, windows, fans, air conditioners) in the environment, work inside an isolette as much as possible and minimize opening portholes that allow cold air to flow inside, and warm any oxygen or humidified air that comes in contact with the newborn. Placing a cap on the newborn's head would help minimize heat loss through evaporation. Placing the newborn skin-to-skin with the mother helps to prevent heat loss through conduction. Using a radiant warmer to transport a newborn helps minimize heat loss through radiation.

15. A one-day-old neonate born at 32 weeks' gestation is in the neonatal intensive care unit under a radiant overhead warmer. The nurse assesses the morning axilla temperature as 95 degrees F (35 degrees C). What could explain the assessment finding? A. Conduction heat loss is a problem in the baby. B. The supply of brown adipose tissue is not developed. C. Axillary temperatures are not accurate. D. This is a normal temperature.

Answer: B Rationale: Typically newborns use nonshivering thermogenesis for heat production by metabolizing their own brown adipose tissue. However, this preterm newborn has an inadequate supply of brown fat because he or she left the uterus early before the supply was adequate. Conduction heat loss allows an increased transfer of heat from their bodies to the environment, but there is nothing to substantiate conduction heat loss. Axillary temperatures are accurate and the mode of taking temperatures for neonates.

27. At the breech forceps birth of a 32 weeks' gestation neonate, the nurse notes olygohydramnios with green thick amniotic fluid. The maternal history reveals a mother of Hispanic ethnicity with marked hypertension, who admits to using cocaine daily. Which factor(s) may contribute to meconium aspiration syndrome (MAS)? Select all that apply. A. the preterm pregnancy B. the forceps breech birth C. maternal cocaine use D. maternal hypertension E. Hispanic ethnicity F. oligohydramnios present

Answer: B, C, D, F Rationale: The predisposing factors for meconium aspiration syndrome include postterm pregnancy and breech presentation with forceps. Ethnicity (Pacific Islander, Indigenous Australian, Black African) is a factor. Postterm neonates are at risk for MAS, but preterm neonates are not. Exposure to drugs during pregnancy, especially tobacco and cocaine, predispose the neeonate to MAS. Maternal hypertension and oligohydramnios also contribute to MAS.

30. The nurse reviews the medical record of a woman who has come to the clinic for an evaluation. The client has a history of mitral valve prolapse and is listed as risk class II. During the visit, the woman states, "We want to have a baby, but I know I am at higher risk. But what is my risk, really?" Which response by the nurse would be appropriate? A. "If you do get pregnant, you will need to be seen by a cardiologist every other month for monitoring." B. "Your risk during pregnancy is small, but you should see your cardiologist first before getting pregnant." C. "Your heart disease would put too much strain on your heart if you were to get pregnant." D. "Your pregnancy would be uneventful, but you would need specialized care for labor and birth."

Answer: B Rationale: Typically, a woman with class I or II cardiac disease can go through a pregnancy without major complications. For class I disease, there is no detectable increased risk of maternal mortality and no increase or a mild increase in morbidity. For class II disease, there is a small increased risk of maternal mortality or moderate increase in morbidity and cardiac consultation should occur every trimester. It is best to have the woman see her cardiologist before becoming pregnant. A woman with class III disease needs frequent visits with the cardiac care team throughout pregnancy. There is a significantly increased risk of maternal mortality or severe morbidity and cardiologist consult should occur every other month with prenatal care and delivery occurring at an appropriate level hospital. A woman with class IV disease is typically advised to avoid pregnancy.

10. The nurse places a warmed blanket on the scale when weighing a newborn to minimize heat loss via which mechanism? A. evaporation B. conduction C. convection D. radiation

Answer: B Rationale: Using a warmed cloth diaper or blanket to cover any cold surface, such as a scale, that touches a newborn directly helps to prevent heat loss through conduction. Drying a newborn and promptly changing wet linens, clothes, or diapers help reduce heat loss via evaporation. Keeping the newborn out of a direct cool draft, working inside an isolette as much as possible, and minimizing the opening of portholes help prevent heat loss via convection. Keeping cribs and isolettes away from outside walls, cold windows, and air conditioners and using radiant warmers while transporting newborns and performing procedures will help reduce heat loss via radiation.

9. Which intervention would be most appropriate for the nurse to do when assisting parents who have experienced the loss of their preterm newborn? A. Avoid using the terms "death" or "dying." B. Provide opportunities for them to hold the newborn. C. Refrain from initiating conversations with the parents. D. Quickly refocus the parents to a more pleasant topic.

Answer: B Rationale: When dealing with grieving parents, nurses should provide them with opportunities to hold the newborn if they desire. In addition, the nurse should provide the parents with as many memories as possible, encouraging them to see, touch, dress, and take pictures of the newborn. These interventions help to validate the parents' sense of loss, relive the experience, and attach significance to the meaning of loss. The nurse should use appropriate terminology, such as "dying," "died," and "death," to help the parents accept the reality of the death. Nurses need to demonstrate empathy and to respect the parents' feelings, responding to them in helpful and supportive ways. Active listening and allowing the parents to vent their frustrations and anger help validate the parents' feelings and facilitate the grieving process.

7. A woman pregnant with twins comes to the clinic for an evaluation. While assessing the client, the nurse would be especially alert for signs and symptoms for which potential problem? A. oligohydramnios B. preeclampsia C. post-term labor D. chorioamnionitis

Answer: B Rationale: Women with multiple gestations are at high risk for preeclampsia, preterm labor, polyhydramnios, hyperemesis gravidarum, anemia, and antepartal hemorrhage. There is no association between multiple gestations and the development of chorioamnionitis.

18. A nurse is assessing a newborn who is about 41⁄2 hours old. The nurse would expect this newborn to exhibit which behavior? Select all that apply. A. sleeping B. interest in environmental stimuli C. passage of meconium D. difficulty arousing the newborn E. spontaneous Moro reflexes

Answer: B, C Rationale: The newborn is in the second period of reactivity, which begins as the newborn awakens and shows an interest in environmental stimuli. This period lasts 2 to 8 hours in the normal newborn (Boxwell, 2010). Heart and respiratory rates increase. Peristalsis also increases. Thus, it is not uncommon for the newborn to pass meconium or void during this period. In addition, motor activity and muscle tone increase in conjunction with an increase in muscular coordination. Spontaneous Moro reflexes are noted during the first period of reactivity. Sleeping and difficulty arousing the newborn reflect the period of decreased responsiveness.

15. The nurse is assessing a newborn's eyes. Which findings would the nurse identify as normal? Select all that apply. A. slow blink response B. able to track object to midline C. transient deviation of the eyes D. involuntary repetitive eye movement E. absent red reflex

Answer: B, C, D Rationale: Assessment of the eyes should reveal a rapid blink reflex, ability to track objects to the midline, transient strabismus (deviation or wandering of the eyes independently), searching nystagmus (involuntary repetitive eye movement), and a red reflex.

25. A set of newborn twins has been admitted to the neonatal intensive care unit with the diagnosis of fetal growth restriction (FGR). Which maternal factors would predispose the newborn to this diagnosis? Select all that apply. A. hemoglobin 15 g/dl (150 g/l) B. A1C levels of 8% (0.08) C. heroin use disorder D. blood pressure baseline of 170/90 mm Hg E. age 39 years F. multiple gestation

Answer: B, C, D, E, F Rationale: Assessment of the small-for-gestational-age (SGA) or FGR infant begins by reviewing the maternal history to identify risk factors such as maternal age over 30 years, a substance use disorder, hypertension, multiple gestation. Gestational diabetes or diabetes mellitus is also a factor. Normal A1C level is 5.7% (0.57) for a person without diabetes. Hemoglobin is normal for pregnant woman in third trimester.

17. A neonate is exhibiting signs of neonatal abstinence syndrome. Which findings would confirm this diagnosis? Select all that apply. A. adequate rooting and sucking B. frequent sneezing C. persistant fever D. shrill, high-pitched cry E. hypotonic reflexes F. frequent yawning

Answer: B, C, D, F Rationale: Manifestations of neonatal abstinence syndrome include a shrill, high-pitched cry; persistent fever; frequent yawning; and frequent sneezing. Rather than adequate rooting and sucking, these actions will be frantic in a neonate with abstinence syndrome. In addition, these neonates will have hypertonic muscle tone, not hypotonic reflexes.

16. A pregnant woman is admitted with premature rupture of the membranes. The nurse is assessing the woman closely for possible infection. Which findings would lead the nurse to suspect that the woman is developing an infection? Select all that apply. A. fetal bradycardia B. abdominal tenderness C. elevated maternal pulse rate D. decreased C-reactive protein levels E. cloudy malodorous fluid

Answer: B, C, E Rationale: Possible signs of infection associated with premature rupture of membranes include elevation of maternal temperature and pulse rate, abdominal/uterine tenderness, fetal tachycardia over 160 bpm, elevated white blood cell count and C-reactive protein levels, and cloudy, foul- smelling amniotic fluid.

31. A pregnant woman with chronic hypertension is entering her second trimester. The nurse is providing anticipatory guidance to the woman about measures to promote a healthy outcome. The nurse determines that the teaching was successful based on which client statement(s)? Select all that apply. A. "I will need to schedule follow-up appointments every 2 weeks until I reach 32 weeks' gestation." B. "I should try to lie down and rest on my left side for about an hour each day." C. "I will start doing daily counts of my baby's activity at about 24 weeks' gestation." D. "I will need to have an ultrasound at each visit beginning at 28 weeks' gestation." E. "I should take my blood pressure frequently at home and report any high readings."

Answer: B, C, E Rationale: The woman with chronic hypertension will be seen more frequently (every 2 weeks until 28 weeks' gestation and then weekly until birth) to monitor her blood pressure and to assess for any signs of preeclampsia. At approximately 24 weeks' gestation, the woman will be instructed to document fetal movement. At this same time, serial ultrasounds will be prescribed to monitor fetal growth and amniotic fluid volume. The woman should also have daily periods of rest (1 hour) in the left lateral recumbent position to maximize placental perfusion and use home blood pressure monitoring devices frequently (daily checks would be preferred), reporting any elevations.

24. A pregnant woman at 31-weeks' gestation calls the clinic and tells the nurse that she is having contractions sporadically. Which instructions would be most appropriate for the nurse to give the woman? Select all that apply. A. "Walk around the house for the next half hour." B. "Drink two or three glasses of water." C. "Lie down on your back." D. "Try emptying your bladder." E. "Stop what you are doing and rest."

Answer: B, D, E Rationale: Appropriate instructions for the woman who may be experiencing preterm labor include having the client stop what she is doing and rest for an hour, empty her bladder, lie down on her left side, and drink two to three glasses of water.

19. A couple has just given birth to a baby who has low Apgar scores due to asphyxia from prolonged cord compression. The neonatologist has given a poor prognosis to the newborn, who is not expected to live. Which interventions are appropriate at this time? Select all that apply. A. Advise the parents that the hospital can make the arrangements. B. Offer to pray with the family if appropriate. C. Leave the parents to talk through their next steps. D. Initiate spiritual comfort by calling the hospital clergy, if appropriate. E. Respect variations in the family's spiritual needs and readiness.

Answer: B, D, E Rationale: When assisting the parents to cope with a perinatal loss, the nurse must respect variations in the family's spiritual needs and readiness. The nurse will also initiate spiritual comfort by calling the hospital clergy, if appropriate, and can offer to pray with the family, if appropriate.

15. A neonate born to a mother who was abusing heroin is exhibiting signs and symptoms of withdrawal. Which signs would the nurse assess? Select all that apply. A. low whimpering cry B. hypertonicity C. lethargy D. excessive sneezing E. overly vigorous sucking F. tremors

Answer: B, D, F Rationale: Signs and symptoms of withdrawal, or neonatal abstinence syndrome, include: irritability, hypertonicity, excessive and often high-pitched crying, vomiting, diarrhea, feeding disturbances, respiratory distress, disturbed sleeping, excessive sneezing and yawning, nasal stuffiness, diaphoresis, fever, poor sucking, tremors, and seizures.

29. A nurse suspects that a client may be developing disseminated intravascular coagulation. The woman has a history of placental abruption (abruptio placentae) during birth. Which finding would help to support the nurse's suspicion? A. severe uterine pain B. board-like abdomen C. appearance of petechiae D. inversion of the uterus

Answer: C Rationale: A complication of abruptio placentae is disseminated intravascular coagulation (DIC), which is manifested by petechiae, ecchymoses, and other signs of impaired clotting. Severe uterine pain, a board-like abdomen, and uterine inversion are not associated with DIC and placental abruption.

9. A newborn is suspected of having fetal alcohol syndrome. Which finding would the nurse expect to assess? A. bradypnea B. hydrocephaly C. flattened maxilla D. hypoactivity

Answer: C Rationale: A newborn with fetal alcohol syndrome exhibits characteristic facial features such as microcephaly (not hydrocephaly), small palpebral fissures, and abnormally small eyes, flattened or absent maxilla, epicanthal folds, thin upper lip, and missing vertical groove in the median portion of the upper lip. Bradypnea is not typically associated with fetal alcohol syndrome. Fine and gross motor development is delayed, and the newborn shows poor hand-eye coordination but not hypoactivity.

2. A nurse is assessing a postterm newborn. Which finding would the nurse correlate with this gestational age variation? A. moist, supple, plum skin appearance B. abundant lanugo and vernix C. thin umbilical cord D. absence of sole creases

Answer: C Rationale: A postterm newborn typically exhibits a thin umbilical cord; dry, cracked, wrinkled skin; limited vernix and lanugo; and creases covering the entire soles of the feet.Question format: Multiple Choice

24. A nurse is developing a plan of care for a woman who is at risk for thromboembolism. Which measure would the nurse include as the most cost-effective method for prevention? A. prophylactic heparin administration B. compression stockings C. early ambulation D. warm compresses

Answer: C Rationale: Although compression stockings and prophylactic heparin administration may be appropriate, the most cost-effective preventive method is early ambulation. It is also the easiest method. Warm compresses are used to treat superficial venous thrombosis.

13. A nurse is preparing a presentation for a group of young adult pregnant women about common infections and their effect on pregnancy. When describing the infections, which infection would the nurse include as the most common congenital and perinatal viral infection in the world? A. rubella B. hepatitis B C. cytomegalovirus D. parvovirus B19

Answer: C Rationale: Although rubella, hepatitis B, and parovirus B19 can affect pregnant women and their fetuses, cytomegalovirus (CMV) is the most common congenital and perinatal viral infection in the world. CMV is the leading cause of congenital infection, with morbidity and mortality at birth and sequelae. Each year approximately 1% to 7% of pregnant women acquire a primary CMV infection. Of these, about 30% to 40% transmits infection to their fetuses.

13. A newborn is suspected of developing persistent pulmonary hypertension. The nurse would expect to prepare the newborn for which procedure to confirm the suspicion? A. chest X-ray B. blood cultures C. echocardiogram D. stool for occult blood

Answer: C Rationale: An echocardiogram is used to reveal right-to-left shunting of blood to confirm the diagnosis of persistent pulmonary hypertension. Chest X-ray would be most likely used to aid in the diagnosis of RDS or TTN. Blood cultures would be helpful in evaluating for neonatal sepsis. Stool for occult blood may be done to evaluate for NEC.

21. A group of nurses are reviewing information about the changes in the newborn's lungs that must occur to maintain respiratory function. The nurses demonstrate understanding of this information when they identify which event as occurring first? A. expansion of the lungs B. increased pulmonary blood flow C. initiation of respiratory movement D. redistribution of cardiac output

Answer: C Rationale: Before the newborn's lungs can maintain respiratory function, the following events must occur: respiratory movement must be initiated; lungs must expand, functional residual capacity must be established, pulmonary blood flow must increase, and cardiac output must be redistributed.

15. A nurse is preparing an inservice education program for a group of nurses about dystocia involving problems with the passenger. Which problem would the nurse likely include as the most common? A. macrosomia B. breech presentation C. persistent occiput posterior position D. multifetal pregnancy

Answer: C Rationale: Common problems involving the passenger include occiput posterior position, breech presentation, multifetal pregnancy, excessive size (macrosomia) as it relates to cephalopelvic disproportion (CPD), and structural anomalies. Of these, persistent occiput posterior is the most common malposition, occurring in about 15% of laboring women.

13. The nurse is assessing the skin of a newborn and notes a rash on the newborn's face and chest. The rash consists of small papules and is scattered with no pattern. The nurse interprets this finding as: A. harlequin sign. B. nevus flames. C. erythema toxicum. D. port wine stain.

Answer: C Rationale: Erythema toxicum (newborn rash) is a benign, idiopathic, generalized, transient rash that occurs in up to 70% of all newborns during the first week of life. It consists of small papules or pustules on the skin resembling flea bites. The rash is common on the face, chest, and back. One of the chief characteristics of this rash is its lack of pattern. It is caused by the newborn's eosinophils reacting to the environment as the immune system matures. Harlequin sign refers to the dilation of blood vessels on only one side of the body, giving the newborn the appearance of wearing a clown suit. It gives a distinct midline demarcation, which is described as pale on the nondependent side and red on the opposite, dependent side. Nevus flammeus or port wine stain is a capillary angioma located directly below the dermis. It is flat with sharp demarcations and is purple-red. This skin lesion is made up of mature capillaries that are congested and dilated.

1. The nurse is teaching a pregnant woman with type 1 diabetes about her diet during pregnancy. Which client statement indicates that the nurse's teaching was successful? A. "I'll basically follow the same diet that I was following before I became pregnant." B. "Because I need extra protein, I'll have to increase my intake of milk and meat." C. "Pregnancy affects insulin production, so I'll need to make adjustments in my diet." D. "I'll adjust my diet and insulin based on the results of my urine tests for glucose."

Answer: C Rationale: In pregnancy, placental hormones cause insulin resistance at a level that tends to parallel growth of the fetoplacental unit. Nutritional management focuses on maintaining balanced glucose levels. Thus, the woman will probably need to make adjustments in her diet. Protein needs increase during pregnancy, but this is unrelated to diabetes. Blood glucose monitoring results typically guide therapy.

4. A nurse is reviewing a client's history and physical examination findings. Which information would the nurse identify as contributing to the client's risk for an ectopic pregnancy? A. use of oral contraceptives for 5 years B. ovarian cyst 2 years ago C. recurrent pelvic infections D. heavy, irregular menses

Answer: C Rationale: In the general population, most cases of ectopic pregnancy are the result of tubal scarring secondary to pelvic inflammatory disease. Oral contraceptives, ovarian cysts, and heavy, irregular menses are not considered risk factors for ectopic pregnancy.

32. A nurse is providing education to a woman who is experiencing postpartum hemorrhage and is to receive a uterotonic agent. The nurse determines that additional teaching is needed when the woman identifies which drug as possibly being prescribed as treatment? A. oxytocin B. methylergonovine C. carboprost D. magnesium sulfate

Answer: D Rationale: Magnesium sulfate is during labor as a tocolytic agent to slow or halt preterm labor. It is not be used to treat postpartum hemorrhage. Oxytocin, methylergonovine, and carboprost are drugs used to manage postpartum hemorrhage.

9. A pregnant woman is diagnosed with iron-deficiency anemia and is prescribed an iron supplement. After teaching her about her prescribed iron supplement, which statement indicates successful teaching? A. "I should take my iron with milk." B. "I should avoid drinking orange juice." C. "I need to eat foods high in fiber." D. "I'll call the primary care provider if my stool is black and tarry."

Answer: C Rationale: Iron supplements can lead to constipation, so the woman needs to increase her intake of fluids and high-fiber foods. Milk inhibits absorption and should be discouraged. Vitamin C- containing fluids such as orange juice are encouraged because they promote absorption. Ideally the woman should take the iron on an empty stomach to improve absorption, but many women cannot tolerate the gastrointestinal discomfort it causes. In such cases, the woman should take it with meals. Iron typically causes the stool to become black and tarry; there is no need for the woman to notify her primary care provider.

23. A woman with hyperemesis gravidarum asks the nurse about suggestions to minimize nausea and vomiting. Which suggestion would be most appropriate for the nurse to make? A. "Make sure that anything around your waist is quite snug." B. "Try to eat three large meals a day with less snacking." C. "Drink fluids in between meals rather than with meals." D. "Lie down for about an hour after you eat."

Answer: C Rationale: Suggestions to minimize nausea and vomiting include avoiding tight waistbands to minimize pressure on the abdomen, eating small frequent meals throughout the day, separating fluids from solids by consuming fluids in between meals; and avoiding lying down or reclining for at least 2 hours after eating.

19. The nurse is developing a plan of care for a neonate experiencing symptoms of drug withdrawal. What should be included in this plan? A. Administer glucose between feedings. B. Schedule feedings every 4 to 6 hours. C. Swaddle the infant between feedings. D. Rock horizontally.

Answer: C Rationale: Supportive interventions to promote comfort include swaddling, low lighting, gentle handling, quiet environment with minimal stimulation, use of soft voices, pacifiers to promote "self-soothing," frequent small feedings, and vertical rocking, which will soothe the newborn's neurological system.

3. The parents of a preterm newborn being cared for in the neonatal intensive care unit (NICU) are coming to visit for the first time. The newborn is receiving mechanical ventilation, intravenous fluids and medications and is being monitored electronically by various devices. Which action by the nurse would be most appropriate? A. Suggest that the parents stay for just a few minutes to reduce their anxiety. B. Reassure them that their newborn is progressing well. C. Encourage the parents to touch their preterm newborn. D. Discuss the care they will be giving the newborn upon discharge.

Answer: C Rationale: The NICU environment can be overwhelming. Therefore, the nurse should address their reactions and explain all the equipment being used. On entering the NICU, the nurse should encourage the parents to touch, interact, and hold their newborn. Doing so helps to acquaint the parents with their newborn, promotes self-confidence, and fosters parent-newborn attachment. The parents should be allowed to stay for as long as they feel comfortable. Reassurance, although helpful, may be false reassurance at this time. Discussing discharge care can be done later once the newborn's status improves and plans for discharge are initiated.Question format: Multiple Choice

26. A pregnant client with iron-deficiency anemia is prescribed an iron supplement. After teaching the woman about using the supplement, the nurse determines that more teaching is needed based on which client statement? A. "Taking the iron supplement with food will help with the side effects." B. "I will need to avoid coffee and tea when I take this supplement." C. "I will take the iron with milk instead of orange or grapefruit juice." D. "If I happen to miss a dose, I will take it as soon as I remember."

Answer: C Rationale: The pregnant client should take the iron supplement with vitamin C-containing fluids such as orange juice, which will promote absorption, rather than milk, which can inhibit iron absorption. Taking iron on an empty stomach improves its absorption, but many women cannot tolerate the gastrointestinal discomfort it causes. In such cases, the woman is advised to take it with meals. The woman also needs instruction about adverse effects, which are predominantly gastrointestinal and include gastric discomfort, nausea, vomiting, anorexia, diarrhea, metallic taste, and constipation. Taking the iron supplement with meals and increasing intake of fiber and fluids helps overcome the most common side effects. If the woman misses a dose, she should take a dose as soon as she remembers.

1. A client is experiencing postpartum hemorrhage, and the nurse begins to massage her fundus. Which action would be most appropriate for the nurse to do when massaging the woman's fundus? A. Place the hands on the sides of the abdomen to grasp the uterus. B. Use an up-and-down motion to massage the uterus. C. Wait until the uterus is firm to express clots. D. Continue massaging the uterus for at least 5 minutes.

Answer: C Rationale: The uterus must be firm before attempts to express clots are made because application of firm pressure on an uncontracted uterus could lead to uterine inversion. One hand is placed on the fundus and the other hand is placed on the area above the symphysis pubis. Circular motions are used for massage. There is no specified amount of time for fundal massage. Uterine tissue responds quickly to touch, so it is important not to overmassage the fundus.

26. Assessment of a newborn reveals transient tachypnea. The nurse reviews the newborn's medical record. Which factor in the newborn's history would the nurse identify as playing a role in this this condition? A. vaginal birth B. shortened labor C. central nervous system depressant during labor D. maternal hypertension

Answer: C Rationale: Transient tachypnea of the newborn occurs when the fetal liquid in the lungs is removed slowly or incompletely. This can be due to the lack of thoracic squeezing that occurs during a cesarean birth or diminished respiratory effort if the mother received central nervous system depressant medication. Prolonged labor, macrosomia of the fetus, and maternal asthma also have been associated with this condition.

9. A client with hyperemesis gravidarum is admitted to the facility after being cared for at home without success. What would the nurse expect to include in the client's plan of care? A. clear liquid diet B. total parenteral nutrition C. nothing by mouth D. administration of labetalol

Answer: C Rationale: Typically, on admission, the woman with hyperemesis has oral food and fluids withheld to rest the gut and receives parenteral fluids to rehydrate and reduce the symptoms. Once the condition stabilizes, oral intake is gradually increased. Total parenteral nutrition may be used if the client's condition does not improve with several days of bed rest, gut rest, IV fluids, and antiemetics. Labetalol is an antihypertensive agent that may be used to treat gestational hypertension, not hyperemesis.

7. A woman with a history of crack cocaine use disorder is admitted to the labor and birth area. While caring for the client, the nurse notes a sudden onset of fetal bradycardia. Inspection of the abdomen reveals an irregular wall contour. The client also reports acute abdominal pain that is continuous. Which condition would the nurse suspect? A. amniotic fluid embolism B. shoulder dystocia C. uterine rupture D. umbilical cord prolapse

Answer: C Rationale: Uterine rupture is associated with crack cocaine use disorder. Generally, the first and most reliable sign is sudden fetal distress accompanied by acute abdominal pain, vaginal bleeding, hematuria, irregular wall contour, and loss of station in the fetal presenting part. Amniotic fluid embolism often is manifested with a sudden onset of respiratory distress. Shoulder dystocia is noted when continued fetal descent is obstructed after the fetal head is delivered. Umbilical cord prolapse is noted as the protrusion of the cord alongside or ahead of the presenting part of the fetus.

20. The health care provider prescribes PGE2 for a woman to help evacuate the uterus following a spontaneous abortion. Which action would be most important for the nurse to do? A. Use clean technique to administer the drug. B. Keep the gel cool until ready to use. C. Maintain the client supine for 30 minutes after administration. D. Administer intramuscularly into the deltoid area.

Answer: C Rationale: When PGE2 is prescribed, the gel should come to room temperature before administering it. Sterile technique should be used, and the client should remain supine for 30 minutes after administration. Rho(D) immune globulin is administered intramuscularly into the deltoid area.

20. A postpartum woman is prescribed oxytocin to stimulate the uterus to contract. Which action would be most important for the nurse to do? A. Administer the drug as an IV bolus injection. B. Give as a vaginal or rectal suppository. C. Piggyback the IV infusion into a primary line. D. Withhold the drug if the woman is hypertensive.

Answer: C Rationale: When giving oxytocin, it should be diluted in a liter of IV solution and the infusion set up to be piggy-backed into a primary line to ensure that the medication can be discontinued readily if hyperstimulation or adverse effects occur. It should never be given as an IV bolus injection. Oxytocin may be given if the woman is hypertensive. Oxytocin is not available as a vaginal or rectal suppository.

11. After teaching a group of nurses working at the women's health clinic about the impact of pregnancy on the older woman, which statement by the group indicates that the teaching was successful? A. "The majority of women who become pregnant over age 35 experience complications." B. "Women over the age of 35 who become pregnant require a specialized type of assessment." C. "Women over age 35 and are pregnant have an increased risk for spontaneous abortions." D. "Women over age 35 are more likely to have a substance use disorder."

Answer: C Rationale: Whether childbearing is delayed by choice or by chance, women starting a family at age 35 or older are not doing so without risk. Women in this age group may already have chronic health conditions that can put the pregnancy at risk. In addition, numerous studies have shown that increasing maternal age is a risk factor for infertility and spontaneous abortions, gestational diabetes, chronic hypertension, postpartum hemorrhage, preeclampsia, preterm labor and birth, multiple pregnancy, genetic disorders and chromosomal abnormalities, placenta previa, fetal growth restriction, low Apgar scores, and surgical births (Dillion et al. 2019). However, even though increased age implies increased complications, most women today who become pregnant after age 34, have healthy pregnancies and healthy newborns. Nursing assessment of the pregnant woman over age 35 is the same as that for any pregnant woman. Women of this age have the same risk for a substance use disorder as any other age group.

2. A client is diagnosed with gestational hypertension and is receiving magnesium sulfate. The nurse determines that the medication is at a therapeutic level based on which finding? A. urinary output of 20 mL per hour B. respiratory rate of 10 breaths/minute C. deep tendons reflexes 2+ D. difficulty in arousing

Answer: C Rationale: With magnesium sulfate, deep tendon reflexes of 2+ would be considered normal and therefore a therapeutic level of the drug. Urinary output of less than 30 mL, a respiratory rate of less than 12 breaths/minute, and a diminished level of consciousness would indicate magnesium toxicity.

27. A client in her first trimester comes to the clinic for an evaluation. Assessment reveals reports of fatigue, anorexia, and frequent upper respiratory infections. The client's skin is pale and the client is slightly tachycardic. The client also reports drinking about 6 cups of coffee on average each day. A diagnosis of iron-deficiency anemia is suspected. The client is scheduled for laboratory testing and the results are as follows: Hemoglobin 11.5 g/dL (115 g/L) Hematocrit 35% (0.35) Serum iron 32 μg/dL (5.73 μmol/L) Serum ferritin 90 ng/dL (90 μg/L)Which laboratory finding would the nurse correlate with the suspected diagnosis? A. Hemoglobin B. Hematocrit C. Serum iron level D. Serum ferritin level

Answer: D Rationale: Laboratory tests for iron-deficiency anemia usually reveal low hemoglobin (less than 11 g/dL or 110 g/L), low hematocrit (less than 35% or 0.35), low serum iron (less than 30 μg/dL or 5.37 μmol/L), microcytic and hypochromic cells, and low serum ferritin (less than 100 ng/dL or 100 μg/L). The client's hemoglobin, hematocrit, and serum iron levels are borderline low normal, but the client's serum ferritin is below 100 ng/dL (100 μg/L), helping to support the diagnosis.

27. A nurse is providing teaching to a new mother about her newborn's nutritional needs. Which suggestions would the nurse include in the teaching? Select all that apply. A. Supplement with iron if the woman is breastfeeding. B. Provide supplemental water intake with feedings. C. Feed the newborn every 2 to 4 hours during the day. D. Burp the newborn frequently throughout each feeding. E. Use feeding time for promoting closeness.

Answer: C, D, E Rationale: Most newborns are on demand feeding schedules and are allowed to feed when they awaken. When they go home, mothers are encouraged to feed their newborns every 2 to 4 hours during the day and only when the newborn awakens during the night for the first few days after birth. Newborns swallow air during feedings, which causes discomfort and fussiness. Parents can prevent this by burping them frequently throughout the feeding. Feeding is also more than an opportunity to get nutrients into the newborn. It is also a time for closeness and sharing. Iron supplementation is recommended for infants who are bottle-fed. Fluid requirements for the newborn and infant do range from 100 to 150 mL/kg daily. This requirement can be met through breast or bottle feedings. Thus, additional water supplementation is not necessary.

14. A nurse is reviewing the medical record of a pregnant client. The nurse suspects that the client may be at risk for dystocia based on which factors? Select all that apply. A. plan for pudendal block anesthetic use B. multiparity C. short maternal stature D. Body mass index 30.2 E. breech fetal presentation

Answer: C, D, E Rationale: Risk factors for dystocia may include maternal short stature, obesity, hydramnios, uterine abnormalities, fetal malpresentation, cephalopelvic disproportion, overstimulation with oxytocin, maternal exhaustion, ineffective pushing, excessive size fetus, poor maternal positioning in labor, and maternal anxiety and fear

findings would indicate that the neonate is in pain? Select all that apply. A. There is flaccid muscle tone of the affected limb. B. Respiration rate is 52 breaths per minute. C. Heart rate is 180 beats per minutes .D. Oxygen saturation level is 88%. E. The infant has facial grimacing and quivering chin.

Answer: C, D, E Rationale: Suspect pain if the newborn exhibits a sudden high-pitched cry; facial grimace is noted with furrowing of the brow and quivering of the chin with an increase in muscle tone when disturbed. Oxygen desaturation will be noted with an increase in heart rate. Increase in the normal blood pressure, pulse, and respiration are noted.

4. Rapid assessment of a newborn indicates the need for resuscitation. The newborn has copious secretiohs. The newborn is dried and placed under a radiant warmer. Which action would the nurse do next? A. Intubate with an appropriate-sized endotracheal tube. B. Give chest compressions at a rate of 80 times per minute. C. Administer epinephrine intravenously. D. Clear the airway with a bulb syringe.

Answer: D Rationale: After placing the newborn's head in a neutral position, the nurse would clear the airway with a bulb syringe or suction. This is followed by assessment of breathing and bagging if needed, placing a pulse oximeter, ventilating the newborn, assessing the heart rate and giving chest compressions if needed, and then admnistering epinephrine and/or volume expansion if needed.

14. The nurse is assessing the respirations of several newborns. The nurse would notify the health care provider for the newborn with which respiratory rate at rest? A. 38 breaths per minute B. 46 breaths per minute C. 54 breaths per minute D. 68 breaths per minute

Answer: D Rationale: After respirations are established in the newborn, they are shallow and irregular, ranging from 30 to 60 breaths per minute, with short periods of apnea (less than 15 seconds). Thus a newborn with a respiratory rate below 30 or above 60 breaths per minute would require further evaluation.

6. A client who is HIV-positive is in her second trimester and remains asymptomatic. She voices concern about her newborn's risk for the infection. Which statement by the nurse would be most appropriate? A. "You'll probably have a cesarean birth to prevent exposing your newborn." B. "Antibodies cross the placenta and provide immunity to the newborn." C. "Wait until after the infant is born, and then something can be done." D. "Antiretroviral medications are available to help reduce the risk of transmission."

Answer: D Rationale: Drug therapy is the mainstay of treatment for pregnant women infected with HIV. The goal of therapy is to reduce the viral load as much as possible; this reduces the risk of transmission to the fetus. Decisions about the method of birth should be based on the woman's viral load, duration of ruptured membranes, progress of labor, and other pertinent clinical factors. The newborn is at risk for HIV because of potential perinatal transmission. Waiting until after the infant is born may be too late.

17. A nurse is teaching a pregnant woman at risk for preterm labor about what to do if she experiences signs and symptoms. The nurse determines that the teaching was successful when the woman makes which statement? A. "I'll sit down to rest for 30 minutes." B. "I'll try to move my bowels." C. "I'll lie down with my legs raised." D. "I'll drink several glasses of water."

Answer: D Rationale: If the woman experiences any signs and symptoms of preterm labor, she should stop what she is doing and rest for 1 hour, empty her bladder, lie down on her side, drink two to three glasses of water, feel her abdomen and note the hardness of the contraction, and call her health care provider and describe the contraction.

17. A nurse is counseling a pregnant woman with rheumatoid arthritis about medications that can be used during pregnancy. The nurse would emphasize the need to avoid which medication at this time? A. hydroxychloroquine B. nonsteroidal anti-inflammatory drugs C. glucocorticoid D. methotrexate

Answer: D Rationale: Methotrexate is contraindicated during pregnancy. For rheumatoid arthritis, medications are limited to hydroxychloroquine, glucocorticoids, and NSAIDS.

4. A newborn has been diagnosed with a group B streptococcal infection shortly after birth. The nurse understands that the newborn most likely acquired this infection from which cause? A. improper hand washing B. contaminated formula C. nonsterile catheter insertion D. mother's birth canal

Answer: D Rationale: Most often, a newborn develops a group B streptococcus infection during the birthing process when the newborn comes into contact with an infected birth canal. Improper hand washing, contaminated formula, and nonsterile catheter insertion would most likely lead to a late-onset infection, which typically occurs in the nursery due to horizontal transmission.

8. A woman with a history of asthma comes to the clinic for evaluation for pregnancy. The woman's pregnancy test is positive. When reviewing the woman's medication therapy regimen for asthma, which medication would the nurse identify as problematic for the woman now that she is pregnant? A. ipratropium B. albuterol C. salmeterol D. Prednisone

Answer: D Rationale: Oral corticosteroids such as prednisone are not preferred for the long-term treatment of asthma during pregnancy. Inhaled steroids are the choice for maintenance medications to reduce inflammation that leads to bronchospasm. Common ones prescribed include beclomethasone and salmeterol. Rescue agents such as albuterol or ipratropium provide immediate symptomatic relief by reducing acute bronchospasm.

19. A nurse is assessing a newborn and observes the newborn moving his head and eyes toward a loud sound. The nurse interprets this as which behavior? A. habituation B. motor maturity C. social behavior D. orientation

Answer: D Rationale: Orientation refers to the response of newborns to stimuli. It reflects newborns' response to auditory and visual stimuli, demonstrated by their movement of head and eyes to focus on that stimulus. Habituation is the newborn's ability to process and respond to visual and auditory stimuli—that is, how well and appropriately he or she responds to the environment. Habituation is the ability to block out external stimuli after the newborn has become accustomed to the activity. Motor maturity depends on gestational age and involves evaluation of posture, tone, coordination, and movements. These activities enable newborns to control and coordinate movement. When stimulated, newborns with good motor organization demonstrate movements that are rhythmic and spontaneous. Social behaviors include cuddling and snuggling into the arms of the parent when the newborn is held.

6. A pregnant client undergoing labor induction is receiving an oxytocin infusion. Which finding would require immediate intervention?A. fetal heart rate of 150 beats/minute B. contractions every 2 minutes, lasting 45 seconds C. uterine resting tone of 14 mm Hg D. urine output of 20 mL/hour

Answer: D Rationale: Oxytocin can lead to water intoxication. Therefore, a urine output of 20 mL/hour is below acceptable limits of 30 mL/hour and requires intervention. FHR of 150 beats/minute is within the accepted range of 120 to 160 beats/minute. Contractions should occur every 2 to 3 minutes, lasting 40 to 60 seconds. A uterine resting tone greater than 20 mm Hg would require intervention.

28. As part of an in-service program to a group of home health care nurses who care for postpartum women, a nurse is describing postpartum depression. The nurse determines that the teaching was successful when the group identifies that this condition becomes evident at which time after birth of the newborn? A. in the first week B. within the first 2 weeks C. in approximately 1 month D. within the first 6 weeks

Answer: D Rationale: PPD usually has a gradual onset and becomes evident within the first 6 weeks postpartum. Postpartum blues typically manifests in the first week postpartum. Postpartum psychosis usually appears about 3 months after birth of the newborn.

20. A neonate born addicted to cocaine is now being treated with medication for acute neonatal abstinence syndrome. Which medication will be prescribed to relieve withdrawal symptoms? A. meperidine B. adrenalin C. naloxone D. morphine sulphate

Answer: D Rationale: Pharmacologic treatment is warranted if conservative measures are not adequate. Common medications used in the management of newborn withdrawal include an opioid (morphine or methadone) and phenobarbital as a second drug if the opiate does not adequately control symptoms. The other drugs are not used in NAS treatment.

2. The nurse is conducting a class for postpartum women about mood disorders. The nurse describes a transient, self-limiting mood disorder that affects mothers after birth. The nurse determines that the women understood the description when they identify the condition as postpartum: A. depression. B. psychosis. C. bipolar disorder. D. blues.

Answer: D Rationale: Postpartum blues are manifested by mild depressive symptoms of anxiety, irritability, mood swings, tearfulness, increased sensitivity, feelings of being overwhelmed, and fatigue. They are usually self-limiting and require no formal treatment other than reassurance and validation of the woman's experience as well as assistance in caring for herself and her newborn. Postpartum depression is a major depressive episode associated with birth. Postpartum psychosis is at the severe end of the continuum of postpartum emotional disorders. Bipolar disorder refers to a mood disorder typically involving episodes of depression and mania.

23. A nurse is providing care to several pregnant women at different weeks of gestation. The nurse would expect to screen for group B streptococcus infection in the client who is at: A. 16 weeks' gestation. B. 28 weeks' gestation. C. 32 weeks' gestation. D. 36 weeks' gestation.

Answer: D Rationale: Pregnant women between 36 and 37 weeks' gestation should be universally screened for GBS infection during a prenatal visit and if positive, receive appropriate intrapartum antibiotic prophylaxis.

1. After spontaneous rupture of membranes, the nurse notices a prolapsed cord. The nurse immediately places the woman in which position? A. supine B. side-lying C. sitting D. knee-chest

Answer: D Rationale: Pressure on the cord needs to be relieved. Therefore, the nurse would position the woman in a modified Sims, Trendelenburg, or knee-chest position. Supine, side-lying, or sitting would not provide relief of cord compression.

18. A nurse is assessing a pregnant woman with gestational hypertension. Which finding would lead the nurse to suspect that the client has developed severe preeclampsia?A. urine protein 300 mg/24 hours B. blood pressure 150/96 mm Hg C. mild facial edema D. hyperreflexia

Answer: D Rationale: Severe preeclampsia is characterized by blood pressure over 160/110 mm Hg, urine protein levels greater than 500 mg/24 hours, and hyperreflexia. Mild facial edema is associated with mild preeclampsia.

8. A nurse is assessing a postpartum client. Which finding would the cause the nurse the greatest concern? A. leg pain on ambulation with mild ankle edema B. calf pain with dorsiflexion of the foot C. perineal pain with swelling along the episiotomy D. sharp, stabbing chest pain with shortness of breath

Answer: D Rationale: Sharp, stabbing chest pain with shortness of breath suggests pulmonary embolism, an emergency that requires immediate action. Leg pain on ambulation with mild edema suggests superficial venous thrombosis. Calf pain on dorsiflexion of the foot may indicate deep vein thrombosis or a strained muscle or contusion. Perineal pain with swelling along the episiotomy might be a normal finding or suggest an infection. Of the conditions, pulmonary embolism is the most urgent.

11. A nurse is assessing a newborn. Which finding would alert the nurse to the possibility of respiratory distress in a newborn? A. symmetrical chest movements B. periodic breathing C. respirations of 40 breaths/minute D. sternal retractions

Answer: D Rationale: Sternal retractions, cyanosis, tachypnea, expiratory grunting, and nasal flaring are signs of respiratory distress in a newborn. Symmetrical chest movements and a respiratory rate between 30 to 60 breaths/minute are typical newborn findings. Some newborns may demonstrate periodic breathing (cessation of breathing lasting 5 to 10 seconds without changes in color or heart rate) in the first few days of life.

28. A pregnant client with preeclampsia is being treated with intravenous magnesium sulfate. The nurse assesses the client's deep tendon reflexes and grades them as 4+. The nurse notifies the health care provider about this finding, describing them using which term to ensure accurate communication? A. Absent B. Average C. Brisk D. Clonus

Answer: D Rationale: The National Institute of Neurological Disorders and Stroke, a division of the National Institutes of Health, published a scale in the early 1990s that, though subjective, is used widely today. It grades reflexes from 0 to 4+. Grades 2+ and 3+ are considered normal, and grades 0 which indicates an absent reflex and 4 which indicates clonus may indicate pathology. Because these are subjective assessments, to improve communication of reflex results, condensed descriptor categories such as absent, average, brisk, or clonus should be used rather than numeric codes. A 4+ grade indicates clonus which is the presence of rhythmic involuntary contractions, most often at the foot or ankle. Sustained clonus confirms central nervous system involvement.

2. A new mother reports that her newborn often spits up after feeding. Assessment reveals regurgitation. Which factor would the nurse integrate into the response? A. newborn being placed prone after feeding B. limited ability of digestive enzymes C. underdeveloped pyloric sphincter D. relaxed cardiac sphincter

Answer: D Rationale: The cardiac sphincter and nervous control of the stomach is immature, which may lead to uncoordinated peristaltic activity and frequent regurgitation. Placement of the newborn is unrelated to regurgitation. Most digestive enzymes are available at birth, but they are limited in their ability to digest complex carbohydrates and fats; this results in fatty stools, not regurgitation. Immaturity of the pharyngoesophageal sphincter and absence of lower esophageal peristaltic waves, not an underdeveloped pyloric sphincter, also contribute to the reflux of gastric contents.

9. A woman who is experiencing postpartum hemorrhage is extremely apprehensive and diaphoretic. The woman's extremities are cool and her capillary refill time is increased. Based on this assessment, the nurse suspects that the client is experiencing approximately how much blood loss? A. 20% B. 30% C. 40% D. 60%

Answer: D Rationale: The client's assessment indicated mild shock, which is associated with a 20% blood loss. Moderate shock occurs with a blood loss of 30 to 40%. Severe shock is associated with a blood loss greater than 40%.

14. A pregnant woman asks the nurse, "I'm a big coffee drinker. Will the caffeine in my coffee hurt my baby?" Which response by the nurse would be most appropriate? A. "The caffeine in coffee has been linked to birth defects." B. "Caffeine has been shown to restrict growth in the fetus." C. "Caffeine is a stimulant and needs to be avoided completely." D. "If you keep your intake to less than 200 mg/day, you should be okay."

Answer: D Rationale: The effect of caffeine intake during pregnancy on fetal growth and development is still unclear. A recent study found that caffeine intake of no more than 200 mg/day during pregnancy does not affect pregnancy duration and the condition of the newborn. Birth defects have not been linked to caffeine consumption, but maternal coffee consumption decreases iron absorption and may increase the risk of anemia during pregnancy. It is not known if there is a correlation between high caffeine intake and miscarriage due to lack of sufficient studies.

14. A preterm newborn is receiving enteral feedings. Which finding would alert the nurse to suspect that the newborn is developing necrotizing enterocolitis [NEC]? A. irritability B. sunken abdomen C. clay-colored stools D. feeding intolerance

Answer: D Rationale: The newborn with NEC may exhibit feeding intolerance with lethargy, abdominal distention and tenderness, and bloody stools.

11. A nurse is assessing a preterm newborn. Which finding would alert the nurse to suspect that a preterm newborn is in pain? A. bradycardia B. oxygen saturation level of 94% C. decreased muscle tone D. sudden high-pitched cry

Answer: D Rationale: The nurse should suspect pain if the newborn exhibits a sudden high-pitched cry, oxygen desaturation, tachycardia, and increased muscle tone.

8. While making rounds in the nursery, the nurse sees a 6-hour-old baby girl gagging and turning bluish. What would the nurse do first? A. Alert the primary care provider stat, and turn the newborn to her right side. B. Administer oxygen via facial mask by positive pressure. C. Lower the newborn's head to stimulate crying. D. Aspirate the oral and nasal pharynx with a bulb syringe.

Answer: D Rationale: The nurse's first action would be to suction the oral and nasal pharynx with a bulb syringe to maintain airway patency. Turning the newborn to her right side will not alleviate the blockage due to secretions. Administering oxygen via positive pressure is not indicated at this time. Lowering the newborn's head would be inappropriate.

6. It is determined that a client's blood Rh is negative and her partner's is Rh positive. To help prevent Rh isoimmunization, the nurse would expect to administer Rho(D) immune globulin at which time? A. at 32 weeks' gestation and immediately before discharge B. 24 hours before birth and 24 hours after birth C. in the first trimester and within 2 hours of birth D. at 28 weeks' gestation and again within 72 hours after birth

Answer: D Rationale: To prevent isoimmunization, the woman should receive Rho(D) immune globulin at 28 weeks and again within 72 hours after birth.

31. A neonate is diagnosed with Erb's palsy after birth. The parents are concerned about their neonate's limp arm. The nurse explains the neonate will be scheduled to receive what recommended treatment for this condition first? A. Physical therapy to the joint and extremity B. Nothing but time and let nature take its course C. Surgery to correct the joint and muscle alignment D. Immobilization of the shoulder and arm

Answer: D Rationale: Treatment for a neonate with Erb palsy usually involves immobilization of the upper arm across the upper abdomen/chest to protect the shoulder from excessive motion for the first week; then gentle passive range-of-motion exercises are performed daily to prevent contractures. Surgery is not needed to regain function since there is no structural injury. Doing nothing will not help the neonate regain function in the extremity.


Kaugnay na mga set ng pag-aaral

Macro Unit 2: Economic Indicators and the Business Cycle

View Set

CHAPTER 1: STRUCTURE & PROPERTIES OF THE ATOM (QUESTIONS)

View Set

Mathematics CKT Sample Test Questions

View Set

Chapter 1 An Introduction to Anatomy and Physiology

View Set

Equal Credit Opportunity Act (ECOA) / Regulation B - CRCM Study Guide

View Set

Chapter 25 Manicuring Final Exam

View Set